[obm-l] Correção e Frações Contí nuas de e e Pi

2006-08-11 Por tôpico claudio\.buffara
Primeiro uma correção:
No problema que eu enviei há pouco, sobre a caminhada na face da Terra, eu só consegui achar uma infinidade enumerável de soluções. Me parece que são as únicas.

***

Alguém saberia explicar porque a fração contínua simples de "e" apresenta uma regularidade enquanto que a de "pi" aparentemente não. Faz sentido dizer que, de alguma forma, "e" é um irracional mais simples do que "pi"? Certamente, a prova da irracionalidade de "e" é bem mais fácil...

e = [2;1,2,1,1,4,1,1,6,1,1,8,1,1,10,1,1,12,1,...]

Pi = [3;7,15,1,292,1,1,1,2,1,3,1,14,2,1,...]

[]s,
Claudio.


Re: [obm-l] Alg. Linear

2006-08-10 Por tôpico claudio\.buffara
Prezado Sr. Marcelo de Moura Costa:

Mandei o e-mail abaixo pra lista obm-l e nao diretamente a V.Sa. O tal e-mail 
foi em resposta a uma mensagem enviada por uma outra 
pessoa, a qual sempre envia problemas pra lista (alguns dos quais bem 
interessantes, eh verdade!) mas raramente comenta ou 
agradece as solucoes que sao enviadas para tais problemas, nunca revela a 
origem destes problemas e nem diz se os estah enviando 
por nao ter conseguido resolve-los ou apenas por acha-los interessantes.

Agora, cah entre nos, V.Sa. se ofende por muito pouco, hem?

[]s,
Claudio.

-- Cabeçalho original ---

De: [EMAIL PROTECTED]
Para: obm-l@mat.puc-rio.br
Cópia: 
Data: Mon, 7 Aug 2006 23:25:12 -0300
Assunto: Re: [obm-l] Alg. Linear

 Sr. Cláudio
 Não entendi o caráter do e-mail a mim enviado, como não o conheço e não
 enviei nenhum problema para a sua pessoa confesso que não entendi com o
 dizer abaixo e principalmente como conseguiu meu e-mail. Sou professor da
 rede pública de Contagem, MG e de pré-vestibular, meu nome é Marcelo de
 Moura . Agradeço uma explicação pelo e-mail.
 Sem mais para o momento.
 Atenciosamente,
 Marcelo de Moura Costa
 
 Em 07/08/06, claudio.buffara [EMAIL PROTECTED] escreveu:
 
  Então você só passa adiante os problemas que te enviam?
  Não tenta resolver antes?
 
*De:* [EMAIL PROTECTED]
*Para:* obm-l@mat.puc-rio.br
*Cópia:*
*Data:* Fri, 4 Aug 2006 15:40:08 + (GMT)
*Assunto:* Re: [obm-l] Alg. Linear
  Deve ser isso mesmo. eh q me passaram, ai num sei se eh aquilo msm.
 
  *Leonnardo Rabello [EMAIL PROTECTED]* escreveu:
 
  Na verdade não é pra mostrar que dada três retas tangentes a uma parábola,
  mostrar que o foco dessa parábola está sobre o circulo circunscrito formado
  pelos 3 pontos de tangência?
  Essa sim, foi questão do IME..
 
 
   Em 01/08/06, Klaus Ferraz [EMAIL PROTECTED] escreveu:
  
Mostre que todo ponto pertencente ao circulo circunscrito a um
   triangulo é foco de uma parabola tangente aos tres lados do triangulo. 
  
   
   --
   Yahoo! Acesso 
   Grátishttp://us.rd.yahoo.com/mail/br/tagline/freeisp/*http://br.acesso.yahoo.com-
Internet rápida e grátis. 
Instale o discador agora!

  
 
 
 
  --
  * Leonnado Rabello
  * linux user #391163
  * msn: [EMAIL PROTECTED]
 
 
   --
  Yahoo! Search
  Música para ver e ouvir: You're Beautiful, do James 
  Blunthttp://us.rd.yahoo.com/mail/br/tagline/search/video/*http://
br.search.yahoo.com/search/video?p=james+bluntei=UTF-8cv=gx=wrtvm=rfr=intl-mail-br-b
 
 
 
 


=
Instruções para entrar na lista, sair da lista e usar a lista em
http://www.mat.puc-rio.br/~nicolau/olimp/obm-l.html
=


[obm-l] Poligonal no Plano

2006-08-10 Por tôpico claudio\.buffara
Quão difícil é este problema?

Considere a seguinte sequência de pontos em R^2:
P_0 = (1,0)
P_1 = ponto da curva y = x^2 e vértice do triângulo equilátero P_0P_1P_2 cuja base P_0P_2situa-se sobre o eixo x.
P_2 = terceiro vértice do triângulo equilátero mencionado acima.
Daí em diante, teremos que, para n = 1,P_(2n), P_(2n+1) e P_(2n+2) serão vértices de triângulos equiláteros cujas bases (P_(2n)P_(2n+2)) situam-se sobre o eixo x e cujo terceiro vértice (P_(2n+1)) situa-se sobre a curva y = x^2.
Calcule o comprimento da poligonal P_0P_1P_2P_3P_n, quando n tende a infinito.

[]s,
Claudio.


[obm-l] Re: [obm-l] Re:[obm-l] Re: Polinôm io nos inteiros

2006-08-09 Por tôpico claudio\.buffara

Ops! Tens razão - mancada minha...

Aqui vai uma nova tentativa:

Seja k o menor inteiro positivo para o qual existem k inteiros maiores do que 1, livres de quadrados e primos entre si dois-a-dois m_1, ..., m_k tais que: 
raiz(m_k) pertence a Q(raiz(m_1), ..., raiz(m_(k-1))).

***

Lema 1:
N é inteiro  1e livre de quadrados == raiz(N) é irracional

Lema 2:
M, N são inteiros  1,primos entre si e livres de quadrados ==
M*Né 1e livre de quadrados

Estes dois lemas decorrem da definição de "inteiro livre de quadrados" (igual a um produto de primos distintos) e da fatoração única em Z.

***

A partir dos lemas é fácil provar que k = 3.

Para simplificar a notação, ponhamos:
m= m_(k-1), 
n= m_k, 
F= Q(raiz(m_1), ..., raiz(m_(k-2))), 
de forma que: 
Q(raiz(m_1), ..., raiz(m_(k-1))) = F(raiz(m))

Pela escolha de k, raiz(n) pertence a F(raiz(m)) mas não pertence a F.
Logo, existem a e b em F tais que raiz(n) = a + b*raiz(m) ==
n = a^2 + b^2m + 2ab*raiz(m) (*)

b = 0 ==
raiz(n) = a = elemento de F ==
contradição ==
b  0

a = 0 ==
raiz(n) = b*raiz(m) ==
(multiplicando ambos os membros por raiz(m) )
raiz(mn) = bm = elemento de F ==
(usando o lema 2 em mn)
m_1, ..., m_(k-2),mn são k-1 inteirosmaiores do que 1 e livres de quadrados tais que: 
raiz(mn) pertence a F = Q(raiz(m_1), ..., raiz(m_(k-2))) ==
contradição à minimalidade de k ==
a  0

Logo, devemos ter ab  0.
Mas, nesse caso, (*) implica que:
raiz(m) = (n- a^2- b^2m)/(2ab) = elemento de F ==
contradição à minimalidade de k.

Em suma, não existe um tal k e o resultado está provado.

Que tal?

***

Sobre o outro problema, eu fiz o seguinte:

Se a = 1, o problem é trivial: todo n divide a^n - 1.
Se a  1 e n é um composto tal que a^n == 1 (mod n),
então:
n divide a^n - 1 == a^n - 1 = n*k, para algum k inteiro
Além disso, como n é composto, a^n - 1 é composto e  n.
a^(a^n - 1) - 1 = a^(nk) - 1 = (a^n)^k - 1 = múltiplo de a^n - 1 ==
a^(a^n - 1) == 1 (mod a^n - 1).
Logo, basta provar que, para cada a  1, existe um composto n tal que: 
a^n == 1 (mod n), pois este n gerará uma sequência infinita de compostos que satisfazem ao enunciado: n, a^n - 1, a^(a^n - 1) - 1, ...

[]s,
Claudio.





De:
[EMAIL PROTECTED]




Para:
obm-l@mat.puc-rio.br




Cópia:





Data:
Tue, 8 Aug 2006 20:03:57 -0300




Assunto:
Re: [obm-l] Re:[obm-l] Re: Polinômio nos inteirosOk, Cláudio, o PROBLEMA 1 está ok, é bem curta a solução, legal.Acho que no PROBLEMA 2, cometeste um engano, na parte:"...
 Suponhamos que k = 2.
 Nesse caso, o corolário 2 diz que Q(raiz(p_1), ..., raiz(p_k)) é uma extensão algébrica finita de Q de grau = 4.
 Como Q tem característica 0, essa extensão é, de fato, simples. Ou seja, existe um real w tal que:
 Q(raiz(p_1), ..., raiz(p_k)) = Q(w).
 (veja qualquer bom livro de álgebra para uma demonstração disso)
 
 Se raiz(p) pertence a Q(w), então vão existir racionais a e b tais que raiz(p) = a + b*w, com b  (caso contrário, raiz(p) seria racional). Elevando ao quadrado:
 p = a^2 + 2abw + b^2w^2 ==
 w é raiz de uma equação do 2o. grau com coeficientes em Q ==
 [Q(w):Q] = 2 ==
 contradição ==
 raiz(p) não pertence a Q(w) = Q(raiz(p_1), ..., raiz(p_k))..."De fato, como K = Q(raiz(p_1), ..., raiz(p_k)) é uma extensão finita do corpo Q, cuja característica é 0, a extensão é separável e finita, logo pelo teorema do elemento primitivo, existe um w tal que K = Q(w). Em seguida, tu supôs por absurdo, que raiz(p) pertence a K e conclui que ele é da forma a+bw. Depois concluiu que w satisfaz um polinômio em Q(x) de grau 2, o que seria um absurdo. Só que se raiz(p) pertence a K então ele é da forma a+bw+...+zw^{q-1} onde q é a dimensão da extensão K/Q, que tu já demonstrou ser pelo menos 4.Segue um outro problema.PROBLEMA 3. Dado a0, mostrar que existem infinitos n compostos tais que a^(n-1) == 1 (mod n).Duda


[obm-l] Re:[obm-l] Re: Polinômio nos intei ros

2006-08-08 Por tôpico claudio\.buffara





De:
[EMAIL PROTECTED]




Para:
obm-l@mat.puc-rio.br




Cópia:





Data:
Mon, 7 Aug 2006 18:42:32 -0300




Assunto:
[obm-l] Re: Polinômio nos inteiros
 2006/8/7, Eduardo Casagrande Stabel [EMAIL PROTECTED]:

 Olá, pessoal da lista.Já pensei sobre este problema mas não tive uma boa idéia que me levasse à solução.PROBLEMA 1. Seja f(x) um polinômio de grau n e coeficientes inteiros. Suponha que existe um inteiro m e um primo p de forma que p divide f(m), f(m+1), ..., f(m+n-1) e f(m+n). Prove que qualquer que seja x inteiro p divide p(x).
Olhe para f(x) em Z_p.
Em Z_p, f(x) é um polinômio de grau = n que tem min(p,n+1) raízes distintas.
Logo, f(x) é identicamente nulo em Z_p, ou seja, f(x) é divisível por p para todod inteiro x.
Um outro problema que o Gugu passou num curso de verão e que tenho curiosidade por saber como resolver é o seguinte.PROBLEMA 2. Sejam p_1, p_2, ..., p_{k-1} e p_k primos distintos. Prove que as raízes quadradas destes primos formam um conjunto linearmente independente sobre o corpo dos racionais. De outra forma mais elementar: se a_1RAIZ(p_1) + ... + a_kRAIZ(p_k) = 0, onde cada a_i é racional, então a_i = 0 para todo i.
De fato, dá até pra fazer uma afirmação um pouco mais forte:
se a_0, a_1, ..., a_k são racionais tais que:
a_0 + a_1*raiz(p_1) + ... + a_k*raiz(p_k) = 0
então a_0 = a_1 = ... = a_k = 0.

***

Lema: 
Se p e q são primos distintos, então:
Q(raiz(p)) é uma extensão de grau 2 de Q
e
Q(raiz(p),raiz(q)) é uma extensão de grau 2 de Q(raiz(p))

Dem:
Como raiz(p) é irracional, Q é um subcorpo próprio de Q(raiz(p)).
Mas Q(raiz(p)) ~ Q[x]/x^2 - p e x^2 - p é irredutível sobre Q.
Logo, [Q(raiz(p)):Q] = 2.

Se raiz(q) pertencesse a Q(raiz(p)), então teríamos:
raiz(q) =a + b*raiz(p) (*), com a e b racionais.
b = 0 == raiz(q) = a = racional == contradição == b  0
a = 0 == raiz(pq) = bp = racional == contradição == a  0
Logo, ab  0. Elevando (*) ao quadrado, obtemos:
q = a^2 + b^2*p + 2ab*raiz(p) ==
raiz(p) = (q - a^2 - b^2*p)/(2ab) = racional == 
contradição ==
raiz(q) não pertence a Q(raiz(p)) ==
x^2 - q é irredutível sobre Q(raiz(p)) ==
[Q(raiz(p),raiz(q)):Q(raiz(p))] = 2, pois
Q(raiz(p),raiz(q)) ~ Q(raiz(p))[x]/x^2 - q

Corolário 1:
Se p e q são primos distintos, então [Q(raiz(p),raiz(q)):Q] = 4

Corolário 2:
Dados primos distintos p_1, p_2, ..., p_n (n = 2), temos:
[Q(raiz(p_1),raiz(p_2),...,raiz(p_n)):Q] = 4.
(de fato, o grau dessa extensão é 2^n, mas esse resultado mais forte não será necessário)

***


O problema estará resolvido se provarmos o seguinte:
Dados os primos distintos p_1, ..., p_k, se p for um primo distinto de todos eles, então: 
raiz(p) não pertence a Q nem a Q(raiz(p_1), ..., raiz(p_k)).

Naturalmente, raiz(p) é irracional.
Além disso, a demonstração do lema cuidou do caso k = 1.

Suponhamos que k = 2.
Nesse caso, o corolário 2 diz que Q(raiz(p_1), ..., raiz(p_k)) é uma extensão algébrica finita de Q de grau = 4.
Como Q tem característica 0, essa extensão é, de fato, simples. Ou seja, existe um real w tal que:
Q(raiz(p_1), ..., raiz(p_k)) = Q(w).
(veja qualquer bom livro de álgebra para uma demonstração disso)

Se raiz(p) pertence a Q(w), então vão existir racionais a e b tais que raiz(p) = a + b*w, com b  (caso contrário, raiz(p) seria racional). Elevando ao quadrado:
p = a^2 + 2abw + b^2w^2 ==
w é raiz de uma equação do 2o. grau com coeficientes em Q ==
[Q(w):Q] = 2 ==
contradição ==
raiz(p) não pertence a Q(w) = Q(raiz(p_1), ..., raiz(p_k))

Logo, raiz(p) não pode ser expresso como uma combinação linear racional de 1, raiz(p_1), ..., raiz(p_k) (e nem como a razão de duas tais combinações lineares).

Em outras palavras, se existirem racionais a, a_0, a_1, ..., a_k tais que:
a*raiz(p) + a_0*1 + a_1*raiz(p_1) + ... + a_k*raiz(p_k) = 0,
então a = a_0 = a_1 = ... = a_k = 0.

[]s,
Claudio.



Re:[obm-l] Velocidades

2006-08-08 Por tôpico claudio\.buffara





De:
[EMAIL PROTECTED]




Para:
obm-l@mat.puc-rio.br




Cópia:





Data:
Mon, 7 Aug 2006 20:56:48 -0300




Assunto:
[obm-l] Velocidades
 Olá pessoal,
 boa noite.
 Tenho um problema de velocidade que só consegui resolver uma parte, se alguém puder me ajudar, ficaria grato.
 No início de um espaço de 10km, um veículo está a 132km/h, e, no final desse espaço, está a 134km/h. O tempo que o carro levou para percorrer esse espaço foi calculado em três minutos. Isso significa que o carro teve velocidade média de 200km/h.
 1- Qual a menor velocidade máxima possível do veículo nesse percurso?

A menor velocidade máxima é 200 km/h e corresponde a uma aceleração instantânea infinita de 132 a 200em t = 0 e uma desaceleração instantênea infinita de 200 para 134 em t = 3 min.
Obviamente, isso é fisicamente impossível, de modo que 200 km/h é apenas uma cota inferior para a velocidade máxima atingida pelo veículo.

 2- Qual a menor aceleração máxima possível no percurso?

Supondo que a aceleração varie instantaneamente de 0 até um dado valor fixo a em t = 0, permaneça nesse valor até t = 3 min e em t = 3 min caia instantâneamente a zero, teremos:
a*0,05 = 200 == a = 4000 km/h^2.
Da mesma forma que no item 1, este valor de a é apenas uma cota inferior para a aceleração máxima atingida pelo veículo durante o percurso.

[]s,
Claudio.


Re: [obm-l] Alg. Linear

2006-08-07 Por tôpico claudio\.buffara
Então você só passa adiante os problemas que te enviam?
Não tenta resolver antes?





De:
[EMAIL PROTECTED]




Para:
obm-l@mat.puc-rio.br




Cópia:





Data:
Fri, 4 Aug 2006 15:40:08 + (GMT)




Assunto:
Re: [obm-l] Alg. LinearDeve ser isso mesmo. eh q me passaram, ai num sei se eh aquilo msm.Leonnardo Rabello [EMAIL PROTECTED] escreveu:
Na verdade não é pra mostrar que dada três retas tangentes a uma parábola, mostrar que o foco dessa parábola está sobre o circulo circunscrito formado pelos 3 pontos de tangência?Essa sim, foi questão do IME..
 Em 01/08/06, Klaus Ferraz [EMAIL PROTECTED] escreveu:

 Mostre que todo ponto pertencente ao circulo circunscrito a um triangulo é foco de uma parabola tangente aos tres lados do triangulo. 
 
 

Yahoo! Acesso Grátis - Internet rápida e grátis. Instale o discador agora! 

 -- * Leonnado Rabello* linux user #391163* msn: [EMAIL PROTECTED] 


Yahoo! SearchMúsica para ver e ouvir: You're Beautiful, do James Blunt


Re:[obm-l] Equacao

2006-08-03 Por tôpico claudio\.buffara
Aqui vai uma solução razoavelmente feia...

Suponhamos que a equação tenha solução (x,y).

Como n = 3, temos que x^n - y^n = 2^3 - 1^3 = 7  4 == k = 3.

2 aparece com o mesmo expoentena decomposição de x e y pois, caso contrário, dividindo x e y por 2^m (m = menor expoente), ficaríamos com:
2^(k-m) = diferença entre um número par e um ímpar == 
2^(k-m)= 1 necessariamente ==
k = m ==
(x/2^m)^n - (y/2^m)^n = 1 ==
sem solução, pois n = 3 ==
contradição

Assim, podemos supor que x e y são ambos ímpares.

x^n - y^n = (x - y)(x^(n-1) + x^(n-2)y + ... + y^(n-1)) = 2^k ==
x - y = 2^r com r = 1, pois x - y é par e positivo
e
o 2o. termo é uma soma de n parcelas ímpares e igual a 2^(k-r) ==
n é par

Suponhamos que n = 2^p*b, onde p = 1 eb é ímpar.

Se b  1, então, como x^n - y^n é múltiplo de 
x^(b-1) + x^(b-1)y + ... + xy^(b-2) + y^(b-1) =
soma de um número ímpar de parcelas ímpares =
ímpar (e maior do que 1) ==
contradição, pois isso também divide 2^(k-r) ==
b = 1 e, portanto, n = 2^p.

x^(2^p) - y^(2^p) = 2^k ==
(x-y)(x+y)(x^2+y^2)...(x^(2^(p-1))+y^(2^(p-1))) = 2^k ==
x-y = 2^r e x+y = 2^s (1 = r  s) ==
x = 2^r*(2^(s-r) + 1) e y = 2^r*(2^(s-r) - 1) ==
x e y são pares ==
contradição

Conclusão: a equação não possui soluções inteiras positivas.

[]s,
Claudio.





De:
[EMAIL PROTECTED]




Para:
obm-l@mat.puc-rio.br




Cópia:





Data:
Wed, 2 Aug 2006 19:30:32 + (GMT)




Assunto:
[obm-l] Equacao
Sejam k ,n inteiros positivos com n2. Mostre que a equacao x^n-y^n=2^k. Nao possui solucao inteira positiva (x,y).



Yahoo! SearchMúsica para ver e ouvir: You're Beautiful, do James Blunt 


Re:[obm-l] Teoria dos numeros?

2006-08-02 Por tôpico claudio\.buffara





De:
[EMAIL PROTECTED]




Para:
obm-l@mat.puc-rio.br




Cópia:





Data:
Tue, 01 Aug 2006 14:37:56 -0400




Assunto:
[obm-l] Teoria dos numeros?
 Liste todos os pares (m,n) para os quais 2^m + 3^n e um quadrado perfeito.
 
Estou supondo que m e n são inteiros não-negativos.

Por inspeção obtemos as soluções:
m = 0, n = 1 == 2^0 + 3^1 = 4
m = 3, n = 0 == 2^3 + 3^0 = 9
Aliás, estas são as únicas soluções com m = 0 ou n = 0.

Quem conhece o triângulo pitagórico (3,4,5) também acha rápido:
m = 4, n = 2 == 2^4 + 3^2 = 25

Alguns casos podem ser eliminados via congruências.

Por exemplo, se m =1 e n é ímpar, então 2^m + 3^n é ímpar.
Além disso, n ímpar == 3^n == 3 (mod 8). 
Logo:
m = 1 == 2^m + 3^n ==2 + 3 ==5 (mod 8).
m = 2 == 2^m + 3^n == 4+ 3 == 7 (mod 8)
m = 3 == 2^m + 3^n == 0 + 3 == 3 (mod 8)
No entanto,o quadrado de um ímparé sempre == 1 (mod 8).
Conclusão: a única solução com n ímpar é m = 0, n = 1.

***

n é par (n = 2p,p = 0) ==
2^m + 3^(2p) = a^2 ==
2^m = (a - 3^p)(a + 3^p) ==
a - 3^p = 2^k e a + 3^p = 2^(m-k), com m  2k ==
2*3^p = 2^(m-k) - 2^k = 2^k*(2^(m-2k) - 1) ==
3^p = 2^(k-1)*(2^(m-2k) - 1) ==
k = 1 (fatoração única em Z) ==
3^p = 2^(m-2) - 1 ==
m = 3

m = 3 == 3^p = 1 == p = 0 == n = 0 == (m,n) = (3,0)
m = 4 == 3^p = 3 == p = 1 == n = 2 == (m,n) = (4,2)
m = 5 ==
(fazendo q = m-2, de modo que q = 3)
2^q = 3^p + 1 ==
3^p ==-1 == 7(mod 8) ==
não há soluções neste caso, pois 3^p == 1 ou 3 (mod 8), conforme p seja par ou ímpar

Logo, as únicas soluções são (0,1), (3,0) e (4,2).

[]s,
Claudio.


Re: [obm-l] numeros perfeitos

2006-07-31 Por tôpico claudio\.buffara
Soma dos divisores positivos de um quadrado perfeito =
produto de fatores da forma (1 + p + p^2 + ... + p^(2m)), 
onde p eh primo e m eh inteiro positivo.
Logo, cada fator desse produto eh sempre impar.
Isso eh obvio se p = 2. 
Se p eh impar, basta observar que o fator correspondente consiste na soma de um 
numero impar de parcelas impares. Logo eh impar.
Ou seja, a soma dos divisores de um quadrado perfeito eh sempre impar.

[]s,
Claudio.

-- Cabeçalho original ---

De: [EMAIL PROTECTED]
Para: obm-l@mat.puc-rio.br
Cópia: 
Data: Tue, 25 Jul 2006 12:20:49 -0700 (PDT)
Assunto: Re: [obm-l] numeros perfeitos

 Um número perfeito tem soma de seus divisores
 positivos par; tente provar que tal soma para
 quadrados perfeitos é ímpar.
 
 []'s
 Shine
 
 --- diego andres [EMAIL PROTECTED] wrote:
 
  gostaria de saber como provar que todo numero
  perfeito nunca pode ser quadrado
  perfeito
  
  
na lista, sair da lista e usar a lista em
 http://www.mat.puc-rio.br/~nicolau/olimp/obm-l.html
 =
 
 


=
Instruções para entrar na lista, sair da lista e usar a lista em
http://www.mat.puc-rio.br/~nicolau/olimp/obm-l.html
=


Re:[obm-l] Resultado da IMO 2006

2006-07-29 Por tôpico claudio\.buffara
 
 Godel nao conquistou Medalha Fields mas qualquer historiador serio havera de 
 coloca-lo como um dos Grandes Matematicos do seculo XX enquanto que o 
 Cavalheiro da Rainha, se algum historiador o citar, se muito sera  lembrado 
 como um Matematico mediano e, no entanto, tem Medalha Fields e uma dezena de 
 outras honrarias semelhantes para mostrar.


Oi, Paulo:

Quem eh o Cavalheiro da Rainha? O Michael Attiyah?
E por que ele eh apenas um matematico mediano?

[]s,
Claudio.


=
Instruções para entrar na lista, sair da lista e usar a lista em
http://www.mat.puc-rio.br/~nicolau/olimp/obm-l.html
=


Re:[obm-l] Polinomios

2006-07-28 Por tôpico claudio\.buffara
Como o mestre não respondeu, aqui vai a minha explicação:

Em vez de trocar x por x/3 faça x = y/3.
Daí f(3x-2) = 81f(x) vira f(y-2)=81f(y/3).
Fica mais claro assim?

E o grau foi obtido comparando os termos de maior grau:
f(y-2) = a_n(y-2)^n +... == Termo de maior grau = a_ny^n
81f(y/3)= 81a_n(y/3)^n + ...== T. de m. g. = a_n(81/3^n)y^n
Logo, 81/3^n = 1 == n = 4.

[]s,
Claudio.





De:
[EMAIL PROTECTED]




Para:
obm-l@mat.puc-rio.br




Cópia:





Data:
Tue, 25 Jul 2006 18:43:02 + (GMT)




Assunto:
Re:[obm-l] Polinomios
 Ola mestre,
  nao entendi pq trocou x por x/3 na expressao do polinomio e como q se obteu o grau do polinomio.
 Grato."claudio.buffara" [EMAIL PROTECTED] escreveu:
 1) Determine todos os polinomios P nao identicamente nulos tais que P(3x-2)=81P(x) para todo x real. x = 1 == P(1) = 81P(1) == P(1) = 0 == P(x) = (x - 1)Q(x)P(x - 2) = 81P(x/3)Se P(x) = a_0 + a_1x + ... + a_nx^n, entao, comparando os termos de maior grau:a_nx^n = 81a_n(x/3)^n == n = 4Logo, podemos escrever P(x) = (x - 1)(ax^3 + bx^2 + cx + d)P(x-2) = (x - 3)(a(x-2)^3 + b(x-2)^2 + c(x-2) + d) ==P(x-2) = (x - 3)(ax^3 + (-6a+b)x^2 + (12a-4b+c)x + (-8a+4b-2c+d)81P(x/3) = 81(x/3 - 1)(a(x/3)^3 + b(x/3)^2 + c(x/3) + d) ==81P(x/3) = (x - 3)(ax^3 + 3bx^2 + 9cx + 27d)Igualando coeficientes, teremos:-6a+b = 3b12a-4b+c = 9c-8a+4b-2c+d = 27d ==b = -3ac = 3ad = -a ==P(x) = a(x - 1)(x^3 - 3x^2 + 3x - 1) ==P(x) = a(x - 1)^4onde a = real qualquer nao-nulo.[]s,Claudio.=Instruções para entrar na lista, sair da lista e usar a lista emhttp://www.mat.puc-rio.br/~nicolau/olimp/obm-l.html=


O Yahoo! está de cara nova. Venha conferir!


Re:[obm-l] numeros perfeitos

2006-07-28 Por tôpico claudio\.buffara
Os únicos números perfeitos conhecidos são aqueles da forma:
N = 2^(p-1)*(2^p-1) onde p e 2^p-1 são primos ==
o primo 2^p-1 aparece com expoente 1 na decomposição de N ==
N não pode ser quadrado perfeito.

Para o caso de um número perfeito ímpar (se existir algum...)a conclusão decorredo seguinte resultado, cuja demonstração eu proponho aqui como um exercício:
Se N for um número perfeito ímpar, então N é da forma p^(4k+1)*M^2, onde p é um primo == 1 (mod 4), k é um inteiro não-negativo e M é um inteiro ímpar primo com p.

[]s,
Claudio.





De:
[EMAIL PROTECTED]




Para:
obm-l@mat.puc-rio.br




Cópia:





Data:
Tue, 25 Jul 2006 13:08:17 + (GMT)




Assunto:
[obm-l] numeros perfeitosgostaria de saber como provar que todo numero perfeito nunca pode ser quadrado perfeito


O Yahoo! está de cara nova. Venha conferir!


Re:[obm-l] Polinomios

2006-07-25 Por tôpico claudio\.buffara

 1) Determine todos os polinomios P nao identicamente nulos tais que 
 P(3x-2)=81P(x) para todo x real. 

x = 1 == P(1) = 81P(1) == P(1) = 0 == P(x) = (x - 1)Q(x)
P(x - 2) = 81P(x/3)
Se P(x) = a_0 + a_1x + ... + a_nx^n, entao, comparando os termos de maior grau:
a_nx^n = 81a_n(x/3)^n == n = 4
Logo, podemos escrever P(x) = (x - 1)(ax^3 + bx^2 + cx + d)

P(x-2) = (x - 3)(a(x-2)^3 + b(x-2)^2 + c(x-2) + d) ==
P(x-2) = (x - 3)(ax^3 + (-6a+b)x^2 + (12a-4b+c)x + (-8a+4b-2c+d)
 
81P(x/3) = 81(x/3 - 1)(a(x/3)^3 + b(x/3)^2 + c(x/3) + d) ==
81P(x/3) = (x - 3)(ax^3 + 3bx^2 + 9cx + 27d)

Igualando coeficientes, teremos:
-6a+b = 3b
12a-4b+c = 9c
-8a+4b-2c+d = 27d ==

b = -3a
c = 3a
d = -a ==

P(x) = a(x - 1)(x^3 - 3x^2 + 3x - 1) ==
P(x) = a(x - 1)^4
onde a = real qualquer nao-nulo.


[]s,
Claudio.




=
Instruções para entrar na lista, sair da lista e usar a lista em
http://www.mat.puc-rio.br/~nicolau/olimp/obm-l.html
=


[obm-l] Desigualdade com Pi

2006-07-21 Por tôpico claudio\.buffara
Bem, eu estava me referindo a uma demonstracao geometrica ou trigonometrica com um minimo de elegancia (com todo o respeito a sua solucao, claro!) ...

A aproximacao Pi ~ raiz(2) + raiz(3) eh bastante boa. A diferenca eh de apenas 0.00467..., ou seja, menos de 0,15%.
Ao aproximar Pi por excesso por meio do semi-perimetro (ou da area) de umpoligono regular (e convexo) circunscrito ao circulo unitario, esta precisao soh eh ultrapassada quando o numero de lados eh = 48.
Ou seja, 47*tan(Pi/47)  raiz(2) + raiz(3)  48*tan(Pi/48)  Pi.
Isso talvez signifique que uma demonstracao puramente geometrica nao eh muito trivial.

[]s,
Claudio.





De:
[EMAIL PROTECTED]




Para:
obm-l@mat.puc-rio.br




Cópia:





Data:
Sat, 15 Jul 2006 17:33:12 +0200




Assunto:
Re: [obm-l] Casa de Pombos e Desigualdade com Pi
 Viva as férias (até que enfim)
 
 Bom, o seu PCP ainda nao foi, mas pra \pi (estilo "NA MARRA"):
 Eleve ao quadrado (todo mundo é positivo):
 2 + 2 raiz(6) + 3  Pi^2 = 2 raiz(6) = Pi^2 - 5
 
 E mais uma vez (notar que Pi  3 = Pi^2  9  5):
 24  Pi^4 - 10Pi^2 + 25 =
 0  Pi^4 - 10 Pi^2 + 1
 
 Agora calcule as raízes de x^4 - 10x^2 + 1 ...
 x^2 = 5 +/- raiz(25 + 1) = apenas duas raizes, as da raiz positiva do quadrado
 x^2 = 10 + um pouquinho
 
 Agora saiba que Pi = 3.14159265358979... e que raiz(10) = 3.16.. e pronto:
 as raizes do polinômio sao maiores do que +- Pi, e portanto o valor em
 Pi é menor do que zero pois o coeficiente de segundo grau é positivo.
 
 Uma calculadora dá:
 sqrt(2) + sqrt(3) - %pi
 ans = 0.0046717
 
 T+,
 -- 
 Bernardo Freitas Paulo da Costa
 
 
 On 7/15/06, claudio.buffara <[EMAIL PROTECTED]>wrote:
 
  Esse tah me enchendo o saco:
 
  Prove que toda sequencia de 2n-1 inteiros (nao necessariamente distintos)
  possui uma subsequencia de n inteiros cuja soma eh divisivel por n.
 
  ***
 
  Ha alguns meses alguem mandou pra lista o problema de se provar que:
  raiz(2) + raiz(3)  Pi.
  Foi enviada alguma solucao?
 
  []s,
 
  Claudio.
 
 
 =
 Instruções para entrar na lista, sair da lista e usar a lista em
 http://www.mat.puc-rio.br/~nicolau/olimp/obm-l.html
 =
 


[obm-l] Re:[obm-l] Grupos Cíclicos

2006-07-20 Por tôpico claudio\.buffara





De:
[EMAIL PROTECTED]




Para:
obm-l@mat.puc-rio.br




Cópia:





Data:
Wed, 19 Jul 2006 13:22:09 + (GMT)




Assunto:
[obm-l] Grupos Cíclicoscleber vieira [EMAIL PROTECTED] escreveu:

 Olá amigos, gostaria de saber qual a condição necessária para que um determinado elemento de um grupo cíclico possa sergerador ?

a eh gerador do grupo ciclico G == ordem de a = ordem de G

. Pergunto issoafim de resolver o seguintes problemas:
 
 1) Sejam A =a, B = b, C =c e D = d os gruposcíclicos de ordens 6, 8, 12 e 20 respectivamente. Determinar todos os geradores destes grupos.

a gera a == a^r gera a para todo r primo com |a|.

Logo, os geradores do grupo ciclico de ordem 6 sao a e a^5, os do de ordem 8 sao a, a^3, a^5 e a^7, etc...

 
 2) Determinar todos os geradoresdo subgrupo de ordem 6 e do subgrupo de ordem 8 do grupo cíclico de ordem 24.
 
Suponhamos que |a| = 24
Entao, o subgrupo de ordem 6 eh {e,a^4,a^8,a^12,a^16,a^20} e os geradores sao a^4 e a^20.
O subgrupo de ordem 8 eh {e,a^3,a^6,a^9,a^12,a^15,a^18,a^21} e os geradores sao a^3, a^9, a^15 e a^21.

[]s,
Claudio.


Re:[obm-l] Resultado da IMO 2006

2006-07-19 Por tôpico claudio\.buffara
Antes de mais nada, parabens a nossa equipe! 
A meu ver, 6 medalhas de bronze mostram muito mais consistencia do que, por exemplo, 1 ouro, 1 prata e 4 maos abanando...

Eu tambem tenho a impressao (por favor me corrijam se eu estiver enganado) de que paises como China e Coreia do Sul preparam seus olimpicos no estilo Kumon, ou seja, fazem cada um deles memorizar centenas (talvez milhares!) de problemas e solucoes para que, na hora da prova, eles dependam mais da memoria do que da criatividade. Isso talvez explique a quantidade de candidatos desses paises que gabaritam as provas da IMO.

Eh claro que as bancas se esforcam pra elaborar problemas queponham a provaa engenhosidade dos candidatos. Mas, como o Gugu me disse uma vez, eh muito facil propor um problema quase impossivel. O dificil eh propor um bom problema que seja resolvivel.Assim, eh possivel que as provas da IMO sejam imperfeitas nesse sentido. Ou seja, se voce tem algum talento matematico (que certamente eh o caso de todos os participantes) e uma preparacao baseada em decoreba intensiva de problemas e metodos de solucao, ha uma boa chance de voce conseguir gabaritar a prova simplesmente por jah ter visto anteriormente alguma questao similar.

[]s,
Claudio.





De:
[EMAIL PROTECTED]




Para:
obm-l@mat.puc-rio.br




Cópia:





Data:
Sun, 16 Jul 2006 08:52:47 -0700 (PDT)




Assunto:
[obm-l] Resultado da IMO 2006
 Oi gente,
 
 Segundo o Mathlinks e mensagens que recebi da equipe
 (via MSN e email) eu tenho a alegria de informá-los
 que toda a equipe do Brasil vai voltar da Eslovênia
 com medalha! Todos ganharam medalha de bronze.
 
 As pontuações são:
 P1 P2 P3 P4 P5 P6 Total
 BRA 1 7 1 0 7 0 0 15
 BRA 2 7 1 0 7 0 0 15
 BRA 3 7 1 1 7 0 0 16
 BRA 4 7 4 0 7 0 0 18
 BRA 5 7 1 1 7 1 0 17
 BRA 6 7 1 0 7 0 0 15
 
 O Brasil ficou em 29o lugar entre os países. Os dez
 primeiros foram (na ordem) China, Rússia, Coréia do
 Sul, Alemanha, EUA, Romênia, Japão, Irã, Moldávia,
 Taiwan.
 
 O Brasil ficou na frente de países como Índia (famosa
 por sua tradição olímpica forte), Suíça, Cazaquistão,
 República Tcheca (que costuma ser forte), boa parte da
 Europa Ocidental (exceções: Reino Unido, Alemanha e
 Itália). Somos o 1o lugar das Américas Central e do
 Sul (o que quer dizer que ganhamos da Argentina,
 hehe). Na América só perdemos para os países da
 América do Norte. Ficamos só 3 posições atrás da
 Bulgária, um país de grande tradição.
 
 Enfim, um resultado que mostra a consistência do
 Brasil na mais importante competição cultural do
 mundo.
 
 Parabéns aos alunos e professores!
 
 []'s
 Shine
 
 PS: Alguém pensou na prova? Vale a pena, é uma das
 melhores IMOs dos últimos anos!
 
 __
 Do You Yahoo!?
 Tired of spam? Yahoo! Mail has the best spam protection around 
 http://mail.yahoo.com 
 =
 Instruções para entrar na lista, sair da lista e usar a lista em
 http://www.mat.puc-rio.br/~nicolau/olimp/obm-l.html
 =
 


Re:[obm-l] Matrizes

2006-07-14 Por tôpico claudio\.buffara





De:
[EMAIL PROTECTED]




Para:
obm-l@mat.puc-rio.br




Cópia:





Data:
Thu, 13 Jul 2006 01:47:19 + (GMT)




Assunto:
[obm-l] Matrizes
 a)Se A é uma matriz de ordem n tal que A^3=4A. Mostre que A+I é inversivel.

Solucao pelo metodo "eu sou burro mas nao sou cego":
Como A^3 - 4A = 0, o polinomio minimo de A tem grau = 3.
Logo, vale a pena procurar uma inversa para A+I da forma:
xA^2 + yA + zI, jah que termos da forma A^k comk = 3 podem ter seu grau reduzido em virtude da relacao A^3 = 4A.
(A+I)(xA^2+yA+zI) = I ==
xA^3 + (x+y)A^2 + (y+z)A + zI = I ==
(x+y)A^2 + (y+z+4x)A + zI = I ==
x+y = 0; y+z+4x = 0; z = 1 ==
x = -1/3; y = 1/3; z = 1 ==
(A+I)^(-1) = (-1/3)A^2 + (1/3)A + I

***

 b)Se A é uma matriz de ordem n tal que A^2p - A^(p+1)=3A, onde p é natural. Mostre que A+I é inversivel.

A ideia aqui eh mostrar que nenhum autovalor de A+I eh igual a 0.

Seja k um autovalor (possivelmente complexo) de A+I ==
existe v em C^n tal que (A+I)v = kv ==
Av = (k-1)v ==
k-1 eh um autovalor de A, associado a v.

Assim, 0 eh autovalor de A+I== -1 eh autovalor de A.

Mas A eh raiz do polinomio f(x) = x^(2p) - x^(p+1) - 3x==
o polinomio minimo de A divide f(x) ==
cada autovalor de A eh raiz de f(x).

Mas f(-1) =4 - (-1)^(p+1) 0 ==
-1 nao eh raiz de f(x) ==
-1 nao eh autovalor de A ==
0 nao eh autovalor de A+I ==
A+I eh invertivel.

[]s,
Claudio.



[obm-l] Casa de Pombos e Desigualdade com Pi

2006-07-14 Por tôpico claudio\.buffara
Esse tah me enchendo o saco:

Prove que toda sequencia de2n-1 inteiros (nao necessariamente distintos) possui uma subsequencia de n inteiros cuja soma eh divisivel por n.

***

Ha alguns meses alguem mandou pra lista o problema de se provar que: 
raiz(2) + raiz(3)  Pi.
Foienviada alguma solucao?

[]s,
Claudio.



[obm-l] Re:[obm-l] Re:[obm-l] FORMULAS DE REDU ÇÃO

2006-07-10 Por tôpico claudio\.buffara
Tens toda a razao. Bela mancada literaria! Ainda bem que esta é uma lista de matemática. Que me perdoem os barões assinalados...

[]s,
Claudio.





De:
[EMAIL PROTECTED]




Para:
obm-l@mat.puc-rio.br




Cópia:





Data:
Sun, 09 Jul 2006 20:54:05 -0300




Assunto:
[obm-l] Re:[obm-l] FORMULAS DE REDUÇÃO
 Caro Cláudio, o trecho de poema abaixo, "minha terra tem palmeiras/onde 
 canta o sabiá", não pertence a Lusíadas e nem a nenhum outro poema de 
 Camões. Esses versos são de Gonçalvez Dias, em "Canto do Exílio".
 
 "sen(a+b) usa os primeiros 2 versos dos Lusiadas, de Camoes:
 Minha terra tem palmeiras
 Onde canta o sabia
 seno a cosseno b
 seno b cosseno a.
 
 Depois eh soh lembrar que:
 cos(-a) = cos(a);
 sen(-a) = -sen(a);
 cos(a) = sen(90 - a);
 a = (a+b)/2 + (a-b)/2;
 b = (a+b)/2 - (a-b)/2
 
 Abraços!
 
 Pedro Cardoso
 
 _
 Inscreva-se no novo Windows Live Mail beta e seja um dos primeiros a testar 
 as novidades-grátis. Saiba mais: 
 http://www.ideas.live.com/programpage.aspx?versionId=5d21c51a-b161-4314-9b0e-4911fb2b2e6d
 
 =
 Instruções para entrar na lista, sair da lista e usar a lista em
 http://www.mat.puc-rio.br/~nicolau/olimp/obm-l.html
 =
 


Re:[obm-l] Probabilidade - Rolagem de dados

2006-07-10 Por tôpico claudio\.buffara
Use funcoes geratrizes (ou será geradoras?).

Supondo as faces equiprováveis, teremos:

Número de Casos Favoráveis:
Coeficiente de t^z na expansão de (t+t^2+...+t^y)^x

Número de Casos Possíveis: y^x

Assim, por exemplo, comx = 2 dados normais (y = 6), a probabilidade de se ter soma z (2 =z =12) é:

(1/36) * Coeficiente de t^z em(t+t^2+t^3+t^4+t^5+t^6)^2 =
1/36 se z = 2 ou z = 12
1/18 se z = 3 ou z = 11
1/12 se z = 4 ou z = 10
1/9 se z = 5 ou z = 9
5/36 se z = 6 ou z = 8
1/6 se z = 7

[]s,
Claudio.






De:
[EMAIL PROTECTED]




Para:
obm-l@mat.puc-rio.br




Cópia:





Data:
Mon, 10 Jul 2006 01:47:36 -0300




Assunto:
[obm-l] Probabilidade - Rolagem de dados
 Se uma pessoa lança x dados de y faces (numeradas de 1 a y), qual é a chance 
 que ela tem de obter um certo resultado z na soma dos valores obtidos em 
 cada rolagem?
 
 Eu me propus esse problema e não consegui achar uma solução geral - apenas 
 uma específica para cada valor de x, que, pelo que observei, é expressa por 
 uma fórmula de grau x-1. Outro detalhe: essa fórmula só funciona até o valor 
 médio (Vm) da soma, que é sempre o mais provável.
 
 Para x =2 e y = 6, por exemplo, Vm = (2.1 + 2.6)/2 = 7.
 Genericamente, Vm = (x + yx)/2 = x(y+1)/2
 
 Os valores acima de x(y+1)/2 têm chance de ocorrência igual ao número que é 
 tão distante de Vm quanto ele. Para x = 2 e y = 6 (Vm = 7), a chance da 
 soma 9 é igual à chance da soma 5. Genericamente, chamando chance de rolagem 
 da soma n de C(n),
 C(a) para a  x(y+1)/2 = C(x(y+1) - a), o que equivale a dizer que, se a+b = 
 x(y+1), C(a) = C(b).
 
 Bem, mesmo que eu tenha falado só besteira - esse é um medo que tenho! - 
 ainda fica a questão, proposta no início do e-mail, e meus agradecimentos 
 pela atenção.
 
 Pedro Lazéra Cardoso
 



Re:[obm-l]

2006-07-10 Por tôpico claudio\.buffara
Esse enunciado está esquisito pois (a,b) parece ser o domínio de f, g e h, enquanto que uma funcao é sobrejetora sobre o seu contra-domínio, que no caso parece ser o R^3. Não seria, por acaso, "injetora"?

[]s,
Claudio.





De:
[EMAIL PROTECTED]




Para:
obm-l@mat.puc-rio.br




Cópia:





Data:
Sun, 9 Jul 2006 23:35:07 -0300




Assunto:
[obm-l]"Suponhamos que C seja uma curva lisa por trechos dada pela função vetorial r(t) = f(t)i + g(t)j + h(t)k, a = t = b, onde pelo menos uma das f, g, h seja sobrejetora em (a, b). Definiremos a função comprimento de arco por s: s(t) = integral |r'(u)|du no intervalo 'a' até 't' = integral sqrt((df/du)^2 + (dg/du)^2 + (dh/du)^2)du no intervalo 'a' até 't'Então s(t) é o comprimento da parte de C entre r(a) e r(t)"Sobre essa citação do livro de um livro de cálculo eu gostaria de entender o porque de "pelo menos uma das f, g, h seja sobrejetora em (a, b)". Porque pelo menos uma das f, g, h precisam ser sobrejetoras no intervalo (a, b)? Muito obrigado.-- Denisson"Você nasce sem pedir mas morre sem querer.Aproveite esse intervalo!"


Re:RES: [obm-l] Pontos de acumulacao

2006-07-10 Por tôpico claudio\.buffara
Alguém conseguiu uma demonstração ou um contra-exemplo pra segunda proposição?

Aliás, isso me lembra um problema proposto há meses pelo Paulo Santa Rita.
Definimos duas funções de Partes(R) em Partes(R):
F(X) = Fecho de X
e
C(X) = R - X = Complementar de X.

Assim, F(Q) = R; F((0,1]) = [0,1]; C((0,1]) = (-inf,0]U (1,+inf); etc...

Em geral, temos que: F(F(X)) = F(X) e C(C(X)) = X.

Dado um subconjunto qualquer A_0de R, calculamos, sucessivamente:
A_1 = g(A_0); A_2 = g(A_1); A_3 = g(A_2); etc.
onde g pode ser tanto F quanto C.

Problema: 
1) Prove que, qualquer que seja A_0, existem naturais m, n tais que0 = m  n e A_m = A_n.
2) Qual o maior valor possível de n-m?
3) Exiba um A_0 e uma sequência de A_i's correspondentes ao valor achado em (2).


Por exemplo: A_0 = [0,1)
A_1 = C(A_0) = (-inf,0) U [1,+inf)
A_2 = F(A_1) = (-inf,0] U [1,+inf)
A_3 = C(A_2) = (0,1)
A_4 = F(A_3) = [0,1]
A_5 = C(A_4) = (-inf,0) U (1,+inf)
A_6 = F(A_5) = (-inf,0] U [1,+inf) = A_2 == n - m = 4

Se não me engano, este problema se deve a Kuratowski.

[]s,
Claudio.





De:
[EMAIL PROTECTED]




Para:
obm-l@mat.puc-rio.br




Cópia:





Data:
Mon, 10 Jul 2006 09:58:31 -0300




Assunto:
RES: [obm-l] Pontos de acumulacao
  
  Seja A um conjunto infinito e limitado de R. Entao, A tem pontos de
  acumulacao (T. de Bolzano/Weierstrass). Definamos A_0 = A e seja A_1 o
  conjunto dos pontos de acumulacao de A_0. Seja agora A_2 o conjunto dos
  pontos de acumulacao de A_1. De modo geral, formemos uma sequencia de
  conjuntos em que cada A_k eh o conjunto dos pontos de acumulacao de
 A_(k-1).
  
  
  Algumas questoes que estou tentando responder:
  
  Se A for enumeravel, teremos necessariamente A_k = vazio para algum k?
  
 Seja A_0 = {racionais em [0,1]} = enumeravel ==
 A_1 = A_2 = ... = A_n = ... = [0,1]  vazio
 
  Se, para algum k, A_k for vazio, entao isto implica que A eh enumeravel?
 
  Se A nao for enumeravel, podemos ter A_k = vazio para algum k? 
 
 
 
 Essas duas ultimas sao equivalentes, nao sao?
 
 []s,
 Claudio.
 
 De fato. Uma eh a contrapositiva da outra. Na realidade, as duas primeiras
 eh que constituem proposicoes logicamente distintas. 
 
 Obrigado.
 Artur
 
 
 
 =
 Instruções para entrar na lista, sair da lista e usar a lista em
 http://www.mat.puc-rio.br/~nicolau/olimp/obm-l.html
 =
 
 =
 Instruções para entrar na lista, sair da lista e usar a lista em
 http://www.mat.puc-rio.br/~nicolau/olimp/obm-l.html
 =
 


Re:[obm-l] pontos num plano

2006-07-09 Por tôpico claudio\.buffara
Considere um disco que contem todos os 100 pontos e as (no maximo) Binom(100,2) 
retas determinadas por estes pontos.
Tome um ponto P fora do tal disco e que nao esteja sobre nenhuma das retas 
mencionadas.
Qualquer semi-reta com origem em P contem no maximo um dos 100 pontos. 
Logo, a cada um dos 100 pontos podemos associar, de forma unica, uma semi-reta 
com origem em P e contendo este ponto.
Agora, tome uma reta passando por P tal que cada semi-plano determinado por ela 
contem 50 das semi-retas mencionadas.
Esta eh a reta desejada.

[]s,
Claudio.

-- Cabeçalho original ---

De: [EMAIL PROTECTED]
Para: obm-l@mat.puc-rio.br
Cópia: 
Data: Thu, 6 Jul 2006 15:45:49 -0300
Assunto: [obm-l] pontos num plano

 Existem 100 pontos num plano.Prove que podemos traçar uma reta tal que haja
 exatamente 50 pontos de cada lado da reta.
 
 


=
Instruções para entrar na lista, sair da lista e usar a lista em
http://www.mat.puc-rio.br/~nicolau/olimp/obm-l.html
=


[obm-l] Re:[obm-l] FORMULAS DE REDUÇÃO

2006-07-09 Por tôpico claudio\.buffara
-- Cabeçalho original ---

De: [EMAIL PROTECTED]
Para: obm-l@mat.puc-rio.br
Cópia: 
Data: Fri, 7 Jul 2006 22:41:02 + (GMT)
Assunto: [obm-l] FORMULAS DE REDUÇÃO

 Um amigo me disse que existe uma regra de memorização pra formulas de redução 
 (trigonometria) num livro da Mir (acho que é o 
Solving Problems in algebra and trigonometry). Alguém sabe que regra é essa?

Nao precisa nenhum livro russo...

sen(a+b) usa os primeiros 2 versos dos Lusiadas, de Camoes:
Minha terra tem palmeiras
Onde canta o sabia
seno a cosseno b
seno b cosseno a.

Depois eh soh lembrar que: 
cos(-a) = cos(a);
sen(-a) = -sen(a);
cos(a) = sen(90 - a);
a = (a+b)/2 + (a-b)/2;
b = (a+b)/2 - (a-b)/2

[]s,
Claudio.



=
Instruções para entrar na lista, sair da lista e usar a lista em
http://www.mat.puc-rio.br/~nicolau/olimp/obm-l.html
=


[obm-l] Re:[obm-l] Re: Convergência de S érie

2006-06-29 Por tôpico claudio\.buffara
-- Cabeçalho original ---

De: [EMAIL PROTECTED]
Para: obm-l@mat.puc-rio.br
Cópia: 
Data: Wed, 28 Jun 2006 19:46:42 -0300
Assunto: [obm-l] Re: Convergência de Série

 Também não sei se tá certo... Mas... =/
 
 Ratio Test (Apostol 1 pag 400): (a_n+1 / a_n) - L qdo n- infinito.
 Se L  1, a série converge.
 
 Como Soma (n=1) (a_n)^2 converge, limite de (a_n+1/a_n)^2 quando n
 tende a infinito é menor que 1 - (a_n+1/a_n) quando n tende a
 infinito é menor que 1
 

Voce nao pode afirmar isso. O teste da razao dah apenas uma condicao suficiente 
mas nao necessaria.
Por exemplo, SOMA 1/n diverge e SOMA 1/n^2 converge, mas o limite a(n+1)/a(n) 
eh 1 nos dois casos.

 Ratio Test no segundo somatório:
 
 ((a_n+1/n+1) / (a_n/n)) = (a_n+1/a_n) x (n/n+1) que é menor que 1,
 logo a série converge.
 
 Em 28/06/06, claudio.buffara[EMAIL PROTECTED] escreveu:
  Segue abaixo o problema 43 do cap. 4 do Curso de Análise - vol. 1 do Elon,
  juntamente com a minha solução errada.
  O problema que proponho é: achar o erro na solução e dar uma solução
  correta.
 
  Seja (a_n) uma sequência de números reais.
  Prove que se SOMA(n=1) (a_n)^2 converge, então SOMA(n=1) (a_n)/n também
  converge.
 
  Solução errada:
  Como SOMA(n=1) (a_n)^2 converge, deve existir n_0 tal que se n  n_0 então
  (a_n)^2  1/n, já que a série harmônica diverge.
  Logo, para n = n_0, |a_n| = 1/raiz(n) ==
  a_n/n = |a_n|/n = 1/n^(3/2) ==
  SOMA(n=1) a_n/n converge, pela comparação com a série:
  SOMA(n=1) 1/n^(3/2), que é convergente.
 
  []s,
  Claudio.
 
 
 
 
 -- 
 Aline Oliveira
 
 =
 Instruções para entrar na lista, sair da lista e usar a lista em
 http://www.mat.puc-rio.br/~nicolau/olimp/obm-l.html
 =
 
 


=
Instruções para entrar na lista, sair da lista e usar a lista em
http://www.mat.puc-rio.br/~nicolau/olimp/obm-l.html
=


[obm-l] Re:[obm-l] Re: [obm-l] Condição d e Existência de quadriláteros

2006-06-29 Por tôpico claudio\.buffara
-- Cabeçalho original ---

De: [EMAIL PROTECTED]
Para: obm-l@mat.puc-rio.br
Cópia: 
Data: Thu, 29 Jun 2006 02:00:42 -0300
Assunto: [obm-l] Re: [obm-l] Condição de Existência de quadriláteros

 Cada um dos lados deve ser menor do que a soma dos outros três.
 
Essa condicao eh certamente necessaria, mas serah que tambem eh suficiente?

 Quadrilátero de lados a,b,c e d, e de diagonais x e y.
 ac+bd=x.y (T. Ptolomeu)

O T. de Ptolomeu soh vale pra quadrilateros ciclicos.

 Supondo sem perda de generalidade dcba= como xd e yd, temos 
 x.yd^2=ac+bdd^2
   - Original Message - 
   From: matduvidas48 
   To: obm-l 
   Sent: Wednesday, June 28, 2006 8:56 PM
   Subject: [obm-l] Condição de Existência de quadriláteros
 
 
   Existe alguma condição de existência para se formar um quadrilátero de 
 lados a , b , c e d?
   (ou seja existe alguma desigualdade?)
 
   Agradeço desde já
 


=
Instruções para entrar na lista, sair da lista e usar a lista em
http://www.mat.puc-rio.br/~nicolau/olimp/obm-l.html
=


[obm-l] Re:[obm-l] Re: [obm-l] Convergênci a de Série

2006-06-29 Por tôpico claudio\.buffara
-- Cabeçalho original ---

De: [EMAIL PROTECTED]
Para: obm-l@mat.puc-rio.br
Cópia: 
Data: Wed, 28 Jun 2006 23:24:48 -0300
Assunto: [obm-l] Re: [obm-l] Convergência de Série

 Olá Claudio,
 nao analisei sua demonstracao, mas segue a minha:
 
 Sabemos que: (a_n - 1/n)^2  0, assim: a_n^2 - a_n/n + 1/n^2  0, logo: a_n/n 
  a_n^2 + 1/n^2
 como SOMA(a_n^2) converge e SOMA(1/n^2) converge, entao, sua soma converge.
 pelo teste da comparacao, SOMA(a_n/n) converge.
 

Isso mesmo, sendo que a consequencia de (a_n - 1/n)^2 = 0 eh, de fato:
a_n/n = (1/2)*(a_n^2 + 1/n^2).

Tambem eh interessante notar que a reciproca nao vale. Tome a_n = 1/log(n)^2
Entao, SOMA a_n/n = SOMA 1/(n*log(n)^2) converge (teste da integral) 
mas SOMA a_n^2 = SOMA 1/log(n)^4 diverge.

***

Outro problema interessante do mesmo capitulo do Elon eh: 
Provar que se (a_n) eh decrescente e SOMA a_n converge entao n*a_n - 0.

Mais uma vez a reciproca nao vale. Tome a_n = 1/(n*log(n)).
A condicao de a_n ser monotona tambem eh essencial. Tome a_n = (-1)^(n)/n.

[]s,
Claudio.

 vou analisar agora sua solucao, se eu encontrar o erro mando em outro e-mail.
 
 abraços,
 Salhab
 
   - Original Message - 
   From: claudio.buffara 
   To: obm-l 
   Sent: Wednesday, June 28, 2006 5:46 PM
   Subject: [obm-l] Convergência de Série
 
 
   Segue abaixo o problema 43 do cap. 4 do Curso de Análise - vol. 1 do Elon, 
 juntamente com a minha solução errada.
   O problema que proponho é: achar o erro na solução e dar uma solução 
 correta.
 
   Seja (a_n) uma sequência de números reais.
   Prove que se SOMA(n=1) (a_n)^2 converge, então SOMA(n=1) (a_n)/n também 
 converge.
 
   Solução errada:
   Como SOMA(n=1) (a_n)^2 converge, deve existir n_0 tal que se n  n_0 então 
 (a_n)^2  1/n, já que a série harmônica diverge.
   Logo, para n = n_0, |a_n| = 1/raiz(n) == 
   a_n/n = |a_n|/n = 1/n^(3/2) ==
   SOMA(n=1) a_n/n converge, pela comparação com a série:
   SOMA(n=1) 1/n^(3/2), que é convergente.
 
   []s,
   Claudio.
 
 


=
Instruções para entrar na lista, sair da lista e usar a lista em
http://www.mat.puc-rio.br/~nicolau/olimp/obm-l.html
=


[obm-l] Re:[obm-l] Re: [obm-l] Convergênci a de Série

2006-06-29 Por tôpico claudio\.buffara





De:

[obm-l] Convergência de Série

2006-06-28 Por tôpico claudio\.buffara
Segue abaixoo problema 43 do cap. 4 do Curso de Análise - vol. 1 do Elon, juntamente com a minhasolução errada.
O problema que proponho é: achar o erro na soluçãoe dar uma solução correta.

Seja (a_n) uma sequência de números reais.
Prove que se SOMA(n=1) (a_n)^2 converge, então SOMA(n=1) (a_n)/n também converge.

Solução errada:
Como SOMA(n=1) (a_n)^2 converge, deve existir n_0 tal que se n  n_0 então (a_n)^2  1/n, já que a série harmônica diverge.
Logo, para n = n_0, |a_n| = 1/raiz(n) == 
a_n/n = |a_n|/n = 1/n^(3/2) ==
SOMA(n=1) a_n/n converge, pela comparação com a série:
SOMA(n=1) 1/n^(3/2), que é convergente.

[]s,
Claudio.



Re:[obm-l] Matrizes

2006-06-28 Por tôpico claudio\.buffara





De:
[EMAIL PROTECTED]




Para:
obm-l@mat.puc-rio.br




Cópia:





Data:
Wed, 28 Jun 2006 17:38:31 + (GMT)




Assunto:
[obm-l] Matrizes
 Sejam M e N matrizes do tipo n x n distintas tais que:
 (i)M^3=N^3
 (ii)MN^2=NM^2
 É possível que X = M^2+ N^2 seja inversível?
 
(M-N)*(M^2+N^2) = M^3+ MN^2 - NM^2 - N^3 = 0.
Se X = M^2+N^2 fosse invertível, bastaria multiplicar a equação acima por X^(-1) que teríamos M-N = 0, contrariando a hipótese de M e N serem distintas. Logo, X não pode ser invertível.


 A e B são matrizes de ordem n tais que AB + A + B=0. Prove que AB=BA.
I = 0 + I = AB + A + B + I = (A+I)(B+I) = (B+I)(A+I) = BA + B + A + I.
(a 4a. igualdade decorre de que A+I e B+I são inversas uma da outra ede que inversas comutam)

[]s,
Claudio.


Re: [obm-l] Calculo Numerico.

2006-06-27 Por tôpico claudio\.buffara
Esse número é a única solução real da equação x = cos(x).

Ele aparece pela seguinte razão:
Dado x(1) qualquer, x(2) = cos(x(1)) pertence ao intervalo [-1,1] e, portanto, x(3) = cos(x(2)) pertence a[cos(1),1].

No intervalo [cos(1),1], a função cos(x) é estritamente decrescente, e sua derivada (igual a -sen(x)) é tal que:
-1  -sen(1)  -sen(x)  -sen(cos(1))  0.
Em particular, para x nesse intervalo, |sen(x)| = sen(1)  1.

Pelo TVM, temos que, dados a e b tais que:
cos(1) = a  b = 1, 
existe c pertencente ao intervalo(a,b) tal que:
cos(b) - cos(a) = -sen(c)*(b - a) ==
|cos(b) - cos(a)| = |sen(c)|*|b - a| = sen(1)*|b - a| ==
cos(x) é uma contração em [cos(1),1]==
cos(x) tem um único ponto fixo nesse intervalo, digamos h, tal que cos(h) = h e a relação de recorrência x(n+1) = cos(x(n)) converge para h, qualquer que seja o valor inicial x(1), já que, como vimos acima, x(3) pertence ao intervalo[cos(1),1].

[]s,
Claudio.





De:
[EMAIL PROTECTED]




Para:
obm-l@mat.puc-rio.br




Cópia:





Data:
Tue, 27 Jun 2006 16:49:50 -0300




Assunto:
Re: [obm-l] Calculo Numerico.Apenas por curiosidade, empiricamente deu este numero (um programa Python é bem útil nestas horas :P)0,73908513321516064165531208767387
 Em 27/06/06, [EMAIL PROTECTED] [EMAIL PROTECTED] escreveu:
Favor quem pode me ajudar com esta questão.Em uma calculadora cientifica, ajuste a medida de ângulos como sendo radianose digite aleatoriamente um número qualquer. Pressione a tecla da funçãocosseno varias vezes (no mínimo 20 vezes). Com isso, não importa qual tenha sido o numero digitado aleatoriamente no inicio, a seqüência de númerosque vão aparecendo no visor á medida que a tecla da função cosseno é pressionadaconverge sempre para o mesmo numero.Identifique que número é esse e por qual motivo ele sempre aparece. Obrigado.


[obm-l] Re:[obm-l] Exercício da Eureka

2006-06-23 Por tôpico claudio\.buffara
Sejam:
S = 1 - 1/2 + 1/3 - 1/4 + + 1/(2n-1)- 1/(2n)
e
H = 1 + 1/2 + 1/3 +  + 1/(2n)

Então:
S=H - 2*(1/2 + 1/4 +... + 1/(2n)) = 
H - (1 + 1/2 + ... + 1/n) =
1/(n+1)+ 1/(n+2) + ... + 1/(2n)

[]s,
Claudio.





De:
[EMAIL PROTECTED]




Para:
obm-l@mat.puc-rio.br




Cópia:





Data:
Thu, 22 Jun 2006 18:09:34 -0300




Assunto:
[obm-l] Exercício da Eureka
 
 
 Demonstre que 1 - 1/2 + 1/3 - 1/4+ ...+ 1/199 - 1/199 = 1/101 +
 1/102 +...+ 1/200
 
 Pra galera se distrais pós jogo seleção.
 []'s.


Re: Re:[obm-l]- Integral

2006-06-23 Por tôpico claudio\.buffara
A série 1 + x + x^2/2 + x^3/3! + ... de fato converge para todo x real, mas não uniformemente.

Pra ver isso, observe que:
e^x - 1 - x - x^2/2 - ... - x^(n-1)/(n-1)! = 
x^n/n! + x^(n+1)/(n+1)! + ... x^n/n!  1, 
desde que x  (n!)^(1/n).

[]s,
Claudio.





De:
[EMAIL PROTECTED]




Para:
obm-l@mat.puc-rio.br




Cópia:





Data:
Fri, 23 Jun 2006 04:03:31 -0300




Assunto:
Re: Re:[obm-l]- Integral
 Olá,
 apenas alguns detalhes..
 
 e^(x^4) = somatorio de n=0 até infinito de x^(4n)/n!
 esta serie converge para todo x real, e é uma série de potências, deste modo, ela é uniformemente convergente para todo x real, e podemos dizer
 que o integral da serie é a serie da integral... deste modo, temos:
 
 integral e^(x^4) = somatorio de n=0 até infinito de x^(4n+1)/[n! (4n+1)]
 
 esta integral convergepara todo x real.
 
 abraços,
 Salhab

- Original Message - 

From: Giuliano (stuart) 
To: obm-l 
Sent: Thursday, June 22, 2006 3:13 PM
Subject: Re:[obm-l]- Integral
 

 Bom Dia! 
 sabemos que e^x=somatório de n=0 até infinito de (x^n)/n!
 mas comoô que vc queré
 e^(x^4) =somatório de n=0 até infinito de (x^4n)/n! logo a integral será
 somatório de n=1 até infinito de (x^(4n-1))/(n!*4n) 
 
 
 
 
 
 
  O pessoal, to precisando de uma luz aqui numa questão 
  Qual é a integral de e^(x^4) dx ? 
  isso se essa primitiva realmente existir 
  
  Obrigado 
  
 
 Abraços, 
 Giuliano Pezzolo Giacaglia 
 (Stuart)


[obm-l] Triangulo Prolongado

2006-06-23 Por tôpico claudio\.buffara
Esse é fácil mas não deixa de ser um resultado curioso (e que eu nunca tinha visto antes):

Tome um triângulo qualquer ABC.
Prolongue BC até P (C entre B e P) de modo que CP = BC.
Prolongue CA até Q (A entre C e Q) de modo que AQ = CA.
Prolongue AB até R (B entre A e R) de modo que BR = AB.
Qual a razão entre as áreas de PQR e ABC?

[]s,
Claudio.


[obm-l] Mais Geometria

2006-06-23 Por tôpico claudio\.buffara
Ao invés de prolongar um triângulo, conforme descrito abaixo, prolongue um quadrilátero qualquer (convexo), de forma análoga, ou seja, se o quadrilátero é ABCD, prolongue AB até P, BC até Q, CD até R e DA até S, de modo que AB = BP, BC = CQ, CD = DR e DA = AS. Qual a relação entre as áreas de ABCD e PQRS?

Será que o fenômeno continua para pentágonos, hexágonos, etc...?
E quanto aos polígonos não-convexos?

***

Outro que eu não conhecia:

No triângulo ABC, retângulo em A, seja P o ponto onde a hipotenusa BC tangencia o círculo inscrito. Calcule BP e PC em função de BC = a, AC = b, AB = c e prove que área(ABC) = BP*PC.

[]s,
Claudio.





De:
[EMAIL PROTECTED]




Para:
"obm-l" obm-l@mat.puc-rio.br




Cópia:





Data:
Fri, 23 Jun 2006 14:21:04 -0300




Assunto:
[obm-l] Triangulo Prolongado
 Esse é fácil mas não deixa de ser um resultado curioso (e que eu nunca tinha visto antes):
 
 Tome um triângulo qualquer ABC.
 Prolongue BC até P (C entre B e P) de modo que CP = BC.
 Prolongue CA até Q (A entre C e Q) de modo que AQ = CA.
 Prolongue AB até R (B entre A e R) de modo que BR = AB.
 Qual a razão entre as áreas de PQR e ABC?
 
 []s,
 Claudio.


Re: Re:[obm-l]- Integral

2006-06-23 Por tôpico claudio\.buffara
Use a definição de convergência uniforme.

Suponhamos que SOMA(n=0) f_n(x) convirja para F(x) para todo x real.

Dizer que a convergência não é uniforme significa dizer que: 
existe eps  0 (no caso, eu usei eps = 1) tal que, para todo n inteiro positivo, podemos obter um x real (eu usei x  (n!)^(1/n)) que satisfaça a:
| F(x) - SOMA(n=0) f_n(x) | = eps.

***

Imagino que oteorema a que você se refere sobre séries de potências seja um que diz que a convergência é uniforme em cada INTERVALO COMPACTO contido no seu intervalo de convergência.

[]s,
Claudio.





De:
[EMAIL PROTECTED]




Para:
obm-l@mat.puc-rio.br




Cópia:





Data:
Fri, 23 Jun 2006 15:38:32 -0300




Assunto:
Re: Re:[obm-l]- Integral
 Olá Cláudio,
 agora vc me deixou com algumas duvidas.. hehe
 
 e^x = 1 + x + x^2/2! + x^3/3! + ...
 ela converge absolutamente para todo x real.
 mas a série 1 + x + x^2/2! + x^3/3! + ... é uma série de potências...
 assim, podemos dizer que seu raio de convergência é infinito..
 e tem um teorema que diz que toda serie de potencias converge uniformemente no seu intervalo de convergencia.
 to semmeu livro agora pra escrever o teorema com todos os detalhes.
 mas é no Apostol Vol 2.
 
 entendi sua demonstracao, mas ela nao estaria contradizendo o q escrevi acima?
 
 abraços,
 Salhab
 
 

- Original Message - 
From: claudio.buffara 
To: obm-l 
Sent: Friday, June 23, 2006 1:18 PM
Subject: Re: Re:[obm-l]- Integral
 
 A série 1 + x + x^2/2 + x^3/3! + ... de fato converge para todo x real, mas não uniformemente.
 
 Pra ver isso, observe que:
 e^x - 1 - x - x^2/2 - ... - x^(n-1)/(n-1)! = 
 x^n/n! + x^(n+1)/(n+1)! + ... x^n/n!  1, 
 desde que x  (n!)^(1/n).
 
 []s,
 Claudio.
 




De:
[EMAIL PROTECTED]




Para:
obm-l@mat.puc-rio.br




Cópia:





Data:
Fri, 23 Jun 2006 04:03:31 -0300




Assunto:
Re: Re:[obm-l]- Integral
  Olá,
  apenas alguns detalhes..
  
  e^(x^4) = somatorio de n=0 até infinito de x^(4n)/n!
  esta serie converge para todo x real, e é uma série de potências, deste modo, ela é uniformemente convergente para todo x real, e podemos dizer
  que o integral da serie é a serie da integral... deste modo, temos:
  
  integral e^(x^4) = somatorio de n=0 até infinito de x^(4n+1)/[n! (4n+1)]
  
  esta integral convergepara todo x real.
  
  abraços,
  Salhab
 
 - Original Message - 

 From: Giuliano (stuart) 
 To: obm-l 
 Sent: Thursday, June 22, 2006 3:13 PM
 Subject: Re:[obm-l]- Integral
  
 
  Bom Dia! 
  sabemos que e^x=somatório de n=0 até infinito de (x^n)/n!
  mas comoô que vc queré
  e^(x^4) =somatório de n=0 até infinito de (x^4n)/n! logo a integral será
  somatório de n=1 até infinito de (x^(4n-1))/(n!*4n) 
  
  
  
  
  
  
   O pessoal, to precisando de uma luz aqui numa questão 
   Qual é a integral de e^(x^4) dx ? 
   isso se essa primitiva realmente existir 
   
   Obrigado 
   
  
  Abraços, 
  Giuliano Pezzolo Giacaglia 
  (Stuart)


Re:[obm-l] Algebra: elementos nilpotentes e aneis de integridade

2006-06-19 Por tôpico claudio\.buffara





De:
[EMAIL PROTECTED]




Para:
"OBM-L" obm-l@mat.puc-rio.br




Cópia:





Data:
Fri, 16 Jun 2006 23:49:35 -0300




Assunto:
[obm-l] Algebra: elementos nilpotentes e aneis de integridade
 Pessoal,
 
 Alguém pode, por favor, me dar uma dica de como resolver estes dois
 problemas de álgebra?
 
 1) Mostre que o conjunto dos elementos nilpotentes de um anel
 comutativo A é um subanel de A.
 
Se a, b são tais que a^m = 0 e b^n = 0, então: 
(ab)^(mn) = 0 e (a-b)^(m+n) = 0
Como A é comutativo, (ab)^(mn) = a^(mn)*b^(mn) = (a^m)^n*(b^n)^m = 0^n*0^m = 0 e
(a - b)^(m+n) = SOMA(k=0...m+n) (-1)^k*Binom(m+n,k)*a^(m+n-k)*b^k
Se k = n, então a^(m+n-k) = a^m*a^(n-k) = 0*a^(n-k) = 0
Se k  n, então b^k = b^n*b^(k-n) = 0*b^(k-n) = 0
Logo, todos os termos do somatório se anulam.
 
 2) Prove detalhadamente: Se a é um elemento do anel de integridade A e
 a^2 = 1, então a = 1 ou a = -1.

a^2 = 1 == a^2 - 1 = 0 == (a - 1)*(a + 1) = 0 
Como A é um domínio de integridade, a - 1 = 0 ou a + 1 = 0 == 
a= 1 ou a = -1.

 Aqui minha primeira dúvida é se isso é realmente verdade. No anel Z_3
 (anel dos inteiros módulo 3), por exemplo, que é um anel de
 integridade, o fato de a^2 = 1 não significade de a = 1 ou a = -1 (em
 Z_3, 2^2 é igual a 1).

Mas em Z_3, -1 = 2...

[]s,
Claudio.



Re: [obm-l] Comutadores de Matrizes

2006-06-19 Por tôpico claudio\.buffara
Não. M = ABA^(-1)B^(-1) == MBA = AB

Eu consegui fazer esse pra matrizes 2x2. Minha idéia foi trabalhar com matrizes elementares da forma:
1 a
0 1

1 0
a1

a 0
0 1/a

0 -a
1/a 0

Eu provei que:
i)cada uma delas é igual a um comutador;
ii) cada matriz de determinante 1 é igual a um produto finito de matrizes elementares dos tipos acima.

Acho que dá pra generalizar pro casonxn.

Pra quem se interessar, esse é o problema 19 da seção 2.7 do Topics in Algebra do Herstein.

[]s,
Claudio.





De:
[EMAIL PROTECTED]




Para:
obm-l@mat.puc-rio.br




Cópia:





Data:
Thu, 15 Jun 2006 17:48:03 -0300




Assunto:
Re: [obm-l] Comutadores de MatrizesBem, isto equivale a escreverAMB=BAcerto?Bem, eu nao sei nada de algelin, mas vou estudar um pouco esta eq...
 Em 09/06/06, claudio.buffara [EMAIL PROTECTED] escreveu:


 Um de álgebra linear pra variar...
 
 Prove que, para cada matriz quadrada M com determinante igual a 1, existem matrizes quadradas invertíveis A e B tais que M = A*B*A^(-1)*B^(-1).
 
 []s,
 
 Claudio.
 


[obm-l] Comutadores de Matrizes

2006-06-09 Por tôpico claudio\.buffara
Um de álgebra linear pra variar...

Prove que, para cada matriz quadrada M com determinante igual a 1, existem matrizes quadradas invertíveis A e B tais que M = A*B*A^(-1)*B^(-1).

[]s,
Claudio.



Re: [obm-l] (2m)!(2n)!/(m!n!(m+n)!)

2006-06-07 Por tôpico claudio\.buffara
Tambémpode ser:
Trinom(2m+2n;m,n,m+n)/Binom(2m+2n,2m),
onde:
Trinom(a+b+c;a,b,c) = (a+b+c)!/(a!*b!*c!)





De:
[EMAIL PROTECTED]




Para:
obm-l@mat.puc-rio.br




Cópia:





Data:
Tue, 06 Jun 2006 11:15:40 -0300




Assunto:
Re: [obm-l] (2m)!(2n)!/(m!n!(m+n)!)
 claudio.buffara wrote:
 
  Alguém conhece algum problema de combinatória cuja resposta seja:
  (2m)!(2n)!/(m!n!(m+n)!) ?
  
  Eu estou tentando provar que este número é inteiro, quaisquer que 
  sejam m e n naturais mediante um argumento combinatório, mas até agora 
  não consegui.
  
  []s,
  Claudio.
  
 
 Oi!
 
 Isso é o mesmo que
 Binom(2m, m)*Binom(2n, n) / Binom(m + n, m)
 isso ajuda alguma interpretação?
 
 PS: sumi por um bom tempo, mas estou vivo!
 
 
 =
 Instruções para entrar na lista, sair da lista e usar a lista em
 http://www.mat.puc-rio.br/~nicolau/olimp/obm-l.html
 =
 


[obm-l] (2m)!(2n)!/(m!n!(m+n)!)

2006-06-02 Por tôpico claudio\.buffara
Alguém conhece algum problema de combinatória cuja resposta seja:
(2m)!(2n)!/(m!n!(m+n)!) ?

Eu estou tentando provar que este número é inteiro, quaisquer que sejam m e n naturais mediante um argumento combinatório, mas até agora não consegui.

[]s,
Claudio.



Re:[obm-l] Primos gemeos

2006-06-01 Por tôpico claudio\.buffara
-- Cabeçalho original ---

De: [EMAIL PROTECTED]
Para: obm-l@mat.puc-rio.br
Cópia: 
Data: Wed, 31 May 2006 19:36:57 -0700 (PDT)
Assunto: [obm-l] Primos gemeos

 Este problema que me foi proposto me pareceu
 interessante:
 
 Mostre que, se a e p forem inteiros positivos com p
 impar, entao o numero 2(a^p - a + 1) nunca estah
 compreendido entre 2 primos gemeos.
 
 Artur
 
 

Como p eh impar a^p - a eh sempre divisivel por 3, pois:
a == 0, 1, 2 (mod 3) == a^p == 0, 1, 2 (mod 3).
Logo, 2(a^p - a) + 3 eh multiplo de 3 e soh serah primo se a^p = a.
Mas nesse caso, 2(a^p - a) + 1 = 1, que nao eh primo.

[]s,
Claudio.



=
Instruções para entrar na lista, sair da lista e usar a lista em
http://www.mat.puc-rio.br/~nicolau/olimp/obm-l.html
=


Re:[obm-l] Existencia de limite

2006-06-01 Por tôpico claudio\.buffara
-- Cabeçalho original ---

De: [EMAIL PROTECTED]
Para: obm-l@mat.puc-rio.br
Cópia: 
Data: Wed, 31 May 2006 20:33:58 -0700 (PDT)
Assunto: [obm-l] Existencia de limite

 A demonstracao do fato a seguir tem, na minha opiniao,
 uns detalhes interessantes. Eh uma extensao do
 criterio de Cauchy a funcoes gerais.
 
 Seja f definida em um subconjunto D de R e com valores
 em R. Se a eh ponto de acumulacao de D, entao f
 apresenta limite real em a se, e e somente se, para
 todo eps 0 existir d 0 tal que, para todos x_1 e x_2
 de D que satisfacam a 0  |x_1 - a|  d e  0  |x_2 -a
 |  d, tivermos |f(x_1) - f(x_2|  eps. A demonstracao
 da parte somente eh imediata, a parte se eh que eh
 mais interessante 
 
 Esta conclusao eh imediatamente extendida para funcoes
 de um espaco metrico em um espaco metrico completo.
 
 Artur
 
  
Oi, Artur:

Ve se isso faz sentido...

Tome uma sequencia (x_n) de elementos de D tal que x_n - a.
Seja (y_n) dada por y_n = f(x_n).

Seja eps  0.
Tomemos o d  0 correspondente, de acordo com a condicao do enunciado.
Como x_n - a, existe N tal que n  N == |x_n - a|  d
Assim, m, n  N == |x_n - a|  d e |x_m - a |  d == |y_m - y_n|  eps.
Logo, (y_n) eh uma sequencia de Cauchy, e portanto, converge, digamos para L.
Em outras palavras, se x_n - a entao y_n = f(x_n) - L.
Como (x_n) foi tomada arbitrariamente (ou seja, eh uma sequencia qualquer com 
limite a), temos que f(x) - L.

[]s,
Claudio.   


=
Instruções para entrar na lista, sair da lista e usar a lista em
http://www.mat.puc-rio.br/~nicolau/olimp/obm-l.html
=


Re: [obm-l] Primos gemeos

2006-06-01 Por tôpico claudio\.buffara
14 está entre 13 e 15, ou pelo menos estava da última vez que eu chequei...





De:
[EMAIL PROTECTED]




Para:
obm-l@mat.puc-rio.br




Cópia:





Data:
Thu, 1 Jun 2006 06:44:43 -0300




Assunto:
Re: [obm-l] Primos gemeos
 Olá Artur,
 
 Posso estar errada, mas para a=2 e p=3 a fórmula falha. Teremos 2(2^3 - 2
 +1) = 2(8-2+1) = 14, que está entre 2 primos gêmeos, a saber 11 e 13.
 
 Helena
 
 - Original Message -
 From: "Artur Costa Steiner" <[EMAIL PROTECTED]>
 To: 
 Sent: Wednesday, May 31, 2006 11:36 PM
 Subject: [obm-l] Primos gemeos
 
 
 Este problema que me foi proposto me pareceu
 interessante:
 
 Mostre que, se a e p forem inteiros positivos com p
 impar, entao o numero 2(a^p - a + 1) nunca estah
 compreendido entre 2 primos gemeos.
 
 Artur
 
 
 
 __
 Do You Yahoo!?
 Tired of spam? Yahoo! Mail has the best spam protection around
 http://mail.yahoo.com
 =
 Instruções para entrar na lista, sair da lista e usar a lista em
 http://www.mat.puc-rio.br/~nicolau/olimp/obm-l.html
 =
 
 
 --
 No virus found in this incoming message.
 Checked by AVG Free Edition.
 Version: 7.1.394 / Virus Database: 268.8.0/352 - Release Date: 30/5/2006
 
 
 =
 Instruções para entrar na lista, sair da lista e usar a lista em
 http://www.mat.puc-rio.br/~nicolau/olimp/obm-l.html
 =
 


Re:[obm-l] somatorio

2006-05-27 Por tôpico claudio\.buffara





De:
[EMAIL PROTECTED]




Para:
obm-l@mat.puc-rio.br




Cópia:





Data:
Sat, 27 May 2006 03:41:49 + (GMT)




Assunto:
[obm-l] somatorio
 Calcule : sum(k=0-n)k^2*C(n,k)*5^k
 
 gab: 5n(5n+1)6^(n-2). 

Usando repetidamente o fato de que k*C(n,k) = n*C(n-1,k-1), temos:
k^2*C(n,k) = 
k*n*C(n-1,k-1) = 
n*(k-1)*C(n-1,k-1) + n*C(n-1,k-1) =
n*(n-1)*C(n-2,k-2) + n*C(n-1,k-1).

Logo, a soma fica:
n*(n-1)*SOMA(k=0...n) C(n-2,k-2)*5^k + 
+ n*SOMA(k=0...n) C(n-1,k-1)*5^k=

n*(n-1)*5^2*SOMA(j=0...n-2) C(n-2,j)*5^j +
n*5*SOMA(j=0...n-1) C(n-1,j)*5^j =

25*n*(n-1)*(1 + 5)^(n-2) + 5*n*(1 + 5)^(n-1) =

(25*n^2 - 25*n + 30*n)*6^(n-2) =

5*n*(5*n+1)*6^(n-2)
[]s,
Claudio.



Re:[obm-l] N. binomial

2006-05-27 Por tôpico claudio\.buffara
Seja x_n = (2+raiz(3))^n + (2-raiz(3))^n.

x_n obedece a uma relacao de recorrencia linear de 2a. ordem, cujo polinomio caracteristico ehx^2 - 4x + 1.

Logo, x_n = 4*x_(n-1) - x_(n-2) com x_0 = 2 e x_1 = 4.

Isso quer dizer que x_n eh sempre par.

Mas 0  2-raiz(3)  1 == 0  (2-raiz(3))^n  1, para n = 1.

Logo, (2+raiz(3))^n  x_n  (2+raiz(3))^n + 1.
Como x_n eh inteiro, soh pode ser x_n = [(2+raiz(3))^n] + 1 ==
[(2+raiz(3))^n] = x_n - 1 eh sempre impar.

[]s,
Claudio.





De:
[EMAIL PROTECTED]




Para:
obm-l@mat.puc-rio.br




Cópia:





Data:
Sat, 27 May 2006 03:23:07 + (GMT)




Assunto:
[obm-l] N. binomialProve que [(2+sqrt(3))^n] é impar para todo n natural. [] detona a parte inteira.





Re: [obm-l] Matriz de Binomiais

2006-05-23 Por tôpico claudio\.buffara
Eu costumo olhar pra determinantes e cofatores apenas em último caso...

MasA é claramente diagonal inferior e a diagonal consiste só de 1's. 
Logo, det(A) = 1 e, portanto, a inversa de A é diagonal inferior com coeficientes inteiros.

Olhando casos pequenos, eu conjecturo que B = A^(-1) é tal que:
b_i,j = (-1)^(i+j)*Binom(i-1,j-1).

Suponhamos que AB = C = (c_i,j). Então:
c_i,j = SOMA(k=1...n) a_i,k*b_k,j =
(-1)^i * SOMA(k=1...n) (-1)^k*Binom(i-1,k-1)*Binom(k-1,j-1) 

Se k  i ou j  k, então o k-ésimo termo da soma é zero.
Logo, podemos supor quej = k = i e ficamos com:

c_i,j = (-1)^i * SOMA(k=j...i) (-1)^k*Binom(i-1,k-1)*Binom(k-1,j-1)
= (-1)^i * SOMA(k=j...i) (-1)^k*((i-1)!*(k-1)!)/((i-k)!*(k-1)!*(k-j)!*(j-1)!)
= (-1)^i * (i-1)!/(j-1)! * SOMA(k=j...i) (-1)^k/((i-k)!*(k-j)!) 
= (-1)^i * (i-1)!/((j-1)!*(i-j)!) * SOMA(k=j...i) (-1)^k * (i-j)!/((i-k)!*(k-j)!)
= (-1)^i *Binom(i-1,j-1) * SOMA(k=0...i-j) (-1)^(k+j) * Binom(i-j,k)
= (-1)^(i+j) * BINOM(i-1,j-1) * SOMA(k=0...i-j) (-1)^k * Binom(i-j,k)
= 0 
(basta ver que a última soma é igual a expansão binomial de (1-1)^(i-j))

Logo, C = I e B é de fato a inversa de A.

Mas, como eu disse, eu estou procurando uma demonstração inteligente deste fato. Afinal, tem que haver uma forma macetosa de se inverter uma matriz cujos coeficientes formam um triângulo de Pascal...

[]s,
Claudio.






De:
[EMAIL PROTECTED]




Para:
obm-l@mat.puc-rio.br




Cópia:





Data:
Tue, 23 May 2006 11:22:17 -0300




Assunto:
Re: [obm-l] Matriz de Binomiais
 Cláudio eu suspeitaria, em princípio que 
 deva existir uma relação de recorrência entre os 
 cofatores dessa matriz para você achar uma relação de inversão
 que se manifeste de forma simples.
 
 Vc conhece alguma relação
 de recorrência simples?
 
 

- Original Message - 
From: claudio.buffara 
To: obm-l 
Sent: Monday, May 22, 2006 1:54 PM
Subject: [obm-l] Matriz de Binomiais
 
 Alguém conhece alguma forma inteligente de se inverter a matriz nxn A = (a_i,j) tal que a_i,j = Binom(i-1,j-1) ?
 
 Obs: Naturalmente, vale a convenção: r  s == Binom(s,r) = 0.
 
 ***
 
 Também estou procurando uma demonstração combinatória de:
 SOMA(k=0...r) (-1)^k*Binom(n,k) = (-1)^r*Binom(n-1,r)
 com 1 = r = n.
 
 []s,
 Claudio.
 


[obm-l] Matriz de Binomiais

2006-05-22 Por tôpico claudio\.buffara
Alguém conhece alguma forma inteligente de se inverter a matriz nxn A = (a_i,j) tal que a_i,j = Binom(i-1,j-1) ?

Obs: Naturalmente, vale a convenção: r  s == Binom(s,r) = 0.

***

Também estou procurando uma demonstração combinatória de:
SOMA(k=0...r) (-1)^k*Binom(n,k) = (-1)^r*Binom(n-1,r)
com 1 = r = n.

[]s,
Claudio.



Re:[obm-l] Euler

2006-05-17 Por tôpico claudio\.buffara
-- Cabeçalho original ---

De: [EMAIL PROTECTED]
Para: obm-l@mat.puc-rio.br
Cópia: 
Data: Tue, 16 May 2006 21:50:00 + (GMT)
Assunto: [obm-l] Euler

 Alguem sabe demonstrar a formula de Euler usando algum argumento combinatorio?
   C(m,0)*C(h,p)+C(m,1)*C(h,p-1)+C(m,2)*C(h,p-2)+..+C(m,p)*C(h,0)=C(m+h,p).

 

Temos m bolas brancas, numeradas de 1 a m, e h bolas pretas, numeradas de 1 a h.
De quantas formas podemos selecionar p bolas dentre estas m+h bolas?

Solucao 1: Binom(m+h,p)

Solucao 2: Numero de conjuntos distintos de p bolas compostos de:
0 bolas brancas e p bolas pretas: Binom(m,0)*Binom(h,p)
1 bola branca e p-1 bolas pretas: Binom(m,1)*Binom(h,p-1)
...
k bolas brancas e p-k bolas pretas: Binom(m,k)*Binom(h,p-k)
...
p bolas brancas e 0 bolas pretas: Binom(m,p)*Binom(h,0)

Somando tudo voce obtem o lado esquerdo.

***

Ou entao, qual o coeficiente de x^p em (x + 1)^(m+h) = (x + 1)^m*(x + 1)^h ?


[]s,
Claudio.




=
Instruções para entrar na lista, sair da lista e usar a lista em
http://www.mat.puc-rio.br/~nicolau/olimp/obm-l.html
=


[obm-l] Re:[obm-l] RES: [obm-l] Re:[obm-l] Re: [obm-l] Problema de Cálculo

2006-05-16 Por tôpico claudio\.buffara
Ops! Voce estah absolutamente certo. Obrigado.

[]s,
Claudio.

-- Cabeçalho original ---

De: [EMAIL PROTECTED]
Para: obm-l@mat.puc-rio.br
Cópia: 
Data: Mon, 15 May 2006 15:37:33 -0300
Assunto: [obm-l] RES: [obm-l] Re:[obm-l] Re: [obm-l] Problema de Cálculo

  
 
 -Mensagem original-
 De: [EMAIL PROTECTED] [mailto:[EMAIL PROTECTED] nome de
 claudio.buffara
 Enviada em: segunda-feira, 15 de maio de 2006 13:06
 Para: obm-l
 Assunto: [obm-l] Re:[obm-l] Re: [obm-l] Problema de Cálculo
 
 
  
 De:[EMAIL PROTECTED]  
 Para:  obm-l@mat.puc-rio.br   
 Cópia:
 Data:  Mon, 15 May 2006 10:49:01 -0300
 Assunto:   [obm-l] Re: [obm-l] Problema de Cálculo
 
  Para quais valores de k a equação e^(2x)=k.sqrt(x) tem exatamente uma
  solução?
  
 Me parece que o problema pode ser refraseado como:
 Pra que valores de k os gráficos de y = e^(2x) e y = k*raiz(x) (x = 0) são
 tangentes?
  
 Suponhamos que eles sejam tangentes em x = a.
 Então, igualando os valores funcionais e as derivadas em x = a, obtemos:
 e^(2a) = k*raiz(a)   e   2*e^(2a) = k/(2*raiz(a)) ==
 2*raiz(a) = 1/(2*raiz(a)) == 
 raiz(a) = 1/4 == 
 [Artur Costa Steiner] 
  
 Tem um engano aqui.  Eh a =1/4. leva a que k = 2*e(1/2) 
  
 a = 1/2 ==
 k = e*raiz(2)
  
 Se k  e*raiz(2), os gráficos não se intersectam e se k  e*raiz(2), eles se
 intersectam em dois pontos.
  
 []s,
 Claudio.
  
 
 
 


=
Instruções para entrar na lista, sair da lista e usar a lista em
http://www.mat.puc-rio.br/~nicolau/olimp/obm-l.html
=


Re:[obm-l] 3 problemas antigos [quase sol. do primeiro]

2006-05-16 Por tôpico claudio\.buffara





De:
[EMAIL PROTECTED]




Para:
obm-l@mat.puc-rio.br




Cópia:





Data:
Mon, 15 May 2006 18:47:05 +




Assunto:
[obm-l] 3 problemas antigos [quase sol. do primeiro]
 Sauda,c~oes,
 
 Aí vai a quase solução do primeiro problema com comentários
 do prof. Rousseau.
 
 Your Download-Link:
 http://rapidshare.de/files/20538502/firstproblem.pdf.html
 
 1) Ache todos os números k naturais tal que
 ( 2^{k-1} - 1 )/ k é um quadrado perfeito.
 
 Ele só conseguiu mostrar que k=3,7 são os únicos primos que
 satisfazem. Mas não que são os únicos naturais.
 

Até porque não são: k = 1 também serve!

[]s,
Claudio.



[obm-l] Re:[obm-l] Re: [obm-l] Problema de C álculo

2006-05-15 Por tôpico claudio\.buffara





De:
[EMAIL PROTECTED]




Para:
obm-l@mat.puc-rio.br




Cópia:





Data:
Mon, 15 May 2006 10:49:01 -0300




Assunto:
[obm-l] Re: [obm-l] Problema de Cálculo

 Para quais valores de k a equação e^(2x)=k.sqrt(x) tem exatamente uma
 solução?
 
Me parece que o problema pode ser refraseado como:
Pra que valores de k os gráficos de y = e^(2x) e y = k*raiz(x) (x = 0) são tangentes?

Suponhamos que eles sejam tangentes em x = a.
Então, igualando os valores funcionais e as derivadas em x = a, obtemos:
e^(2a) = k*raiz(a) e 2*e^(2a) = k/(2*raiz(a)) ==
2*raiz(a) = 1/(2*raiz(a)) == 
raiz(a) = 1/4 == 
a = 1/2 ==
k = e*raiz(2)

Se k  e*raiz(2), os gráficos não se intersectam e se k  e*raiz(2), eles se intersectamem dois pontos.

[]s,
Claudio.



[obm-l] Re: [obm-l] Variável Complexa!

2006-05-15 Por tôpico claudio\.buffara
Nesse caso, nem os reais são necessários. Basta alguns racionais.
Por exemplo, os múltiplos inteiros de 0,01 ou 0,0001.

Quanto ao livro, um quesai mais em conta é o Funções de uma Variável Complexa, do Alcides Lins Neto - Projeto Euclides. R$ 25,00 na SBM.

[]s,
Claudio.





De:
[EMAIL PROTECTED]




Para:
obm-l@mat.puc-rio.br




Cópia:





Data:
Mon, 15 May 2006 09:22:34 -0700 (PDT)




Assunto:
Re: [obm-l] Variável Complexa!
 Umlivro excelente, em Ingles, eh o Complex Analysis,
 do Ahlfors. Pena que um tanto caro.
 
 Alias, eu conheco uma pessoa de nivel, com a qual
 obviamente nao concordo, que vive dizendo que numeros
 complexos nao servem para nada. "Prova": Os numeros
 que medem sua conta bancaria e dinheiro em geral,
 assim como aqueles que medem sua saude, tais como
 indices de colesterol e de glicose no sangue, sao
 expressos em numeros reais. Ele me perguntou, "Vc ja
 oviu alguem dizer que comprou alguma coisa por, por
 exemplo, (200 + 100i) R$?Eh, tem gente assim, mas
 va em frente que o estudo dos complexos eh fascinante.
 
 Artur 
 
 --- Thiago Lucas Castor de Lima <[EMAIL PROTECTED]>
 wrote:
 
  Olá, colegas! Por acaso, vocês tem algum material
  ligado ao cálculo de uma
  variável complexa(apostilas, exercícios, textos em
  geral, indicações de
  livros)? Ficaria muito grato com qualquer sugestão!
  
  Thiago
  
 



Re: [obm-l] duas perguntas!

2006-05-14 Por tôpico claudio\.buffara
E o truque da inducao eh o seguinte:
Suponha spdg que a_1 = a_2 = ... = a_n
Caso 1: se a_1 = a_n, entao, os a_i sao todos iguais a 1 e acabou.
Caso 2: a_1  a_n == a_1  1  a_n == 
(1 - a_1)*(a_n - 1)  0 ==
a_1 + a_n  1 + a_1*a_n ==
(a_2*...*a_(n-1))*(a_1*a_n) = 1 == (pela HI) 
a_2 + .. + a_(n-1) + a_1*a_n = n - 1 ==
a_2 + ... + a_(n-1) + (a_1*a_n + 1) = n ==
a_2 + ... + a(n-1) + (a_1 + a_n) = n

[]s,
Claudio.
-- Cabeçalho original ---

De: [EMAIL PROTECTED]
Para: obm-l@mat.puc-rio.br
Cópia: 
Data: Sat, 13 May 2006 14:44:51 -0300
Assunto: Re: [obm-l] duas perguntas!

 Olá,
 
 a demonstração MA  MG pode ser feita da seguinte maneira, utilizando o 
 seguinte lema:
 
 Seja a_n  0... Se a_1 * a_2 * a_3 * ... * a_n = 1, entao: a_1 + a_2 + a_3 + 
 ... + a_n = n
 
 este lema pode ser demonstrado por inducao.
 
 Deste modo, vamos demonstrar a desigualdade de medias:
 MG = (a_1 * a_2 * a_3 * ... * a_n)^(1/n)
 MG^n = a_1 * a_2 * a_3 * ... * a_n
 (a_1 * a_2 * a_3 * ... * a_n)/(MG^n) = 1
 
 como temos n termos no numerador, e n termos no denominador, podemos escrever 
 do seguinte modo:
 (a_1/MG) * (a_2/MG) * ... * (a_n/MG) = 1
 
 aplicando o teorema:
 
 a_1/MG + a_2/MG + ... + a_n/MG = n
 
 (a_1 + a_2 + ... + a_n)/n = MG
 
 MA = MG
 
 abraços,
 Salhab
 
   - Original Message - 
   From: [EMAIL PROTECTED] 
   To: obm-l@mat.puc-rio.br 
   Sent: Saturday, May 13, 2006 10:31 AM
   Subject: [obm-l] duas perguntas!
 
 
   Bom dia  caros colegas da lista. Tenho duas perguntas a fazer, uma simples 
 e outra nem tanto.
   1. Pode-se dizer que um retângulo ou um quadrado são trapézios, ou melhor, 
 que os paralelogramos são trapézios?
 
   2. Onde eu poderia encontrar uma demonstração não tão complicada sobre a 
 desigualdade entre as médias aritmética e 
geométrica para o caso geral, ou seja, n  1 números positivos?
 
   Um abraço!
 
   Vanderlei
 


=
Instruções para entrar na lista, sair da lista e usar a lista em
http://www.mat.puc-rio.br/~nicolau/olimp/obm-l.html
=


[obm-l] Re: [obm-l] triângulo de área máxima!

2006-05-14 Por tôpico claudio\.buffara
Ou entao, voce pode usar a formula de Heron, juntamente com MG = MA.

Sejam a, b, c os lados e p o semi-perimetro do triangulo.
a  b + c == 2a  a + b + c = 2p == a  p == p-a  0
Analogamente, p-b 0 e p-c  0.
Como p eh constante, maximizar A eh equivalente a maximizar (A^2/p)^(1/3).
Heron == A^2/p = (p-a)(p-b)(p-c)
MG = MA == 
(A^2/p)^(1/3) = ((p-a)(p-b)(p-c))^(1/3) = ((p-a)+(p-b)+(p-c))/3 = p/3 ==
A = p^2/(3*raiz(3)), com igualdade sss p-a = p-b = p-c sss a = b = c

[]s,
Claudio.

- Cabeçalho original ---

De: [EMAIL PROTECTED]
Para: obm-l@mat.puc-rio.br
Cópia: 
Data: Sun, 14 May 2006 06:00:44 -0300
Assunto: Re: [obm-l] triângulo de área máxima!

 On Sat, May 13, 2006 at 03:33:30PM +, [EMAIL PROTECTED] wrote:
  Qual é a forma mais fácil de provar que dado um triângulo com perímetro
  constante, ele terá área máxima quando for equilátero?
 
 Primeiro verifique que dentre os triângulos com base dada (a)
 e soma dos dois outros lados também dada (b+c=2p-a),
 o isósceles (b=c) tem altura (em relação ao lado a) e portanto área
 estritamente maior do que qualquer outro.
 Você pode ver isso observando que, fixando os vértices B e C,
 o LG para o vértice A é uma elipse de focos B e C e o ponto
 mais distante do eixo maior da elipse é a posição desejada de A.
 
 Depois faça o mesmo tipo de raciocínio rodando A, B, C.
 A cada passo, se o triângulo não for equilátero,
 você pode fazer a área ficar maior sem alterar o perímetro.
 Esta seqüência de triângulos tende para o triângulo equilátero.
 
 []s, N.



=
Instruções para entrar na lista, sair da lista e usar a lista em
http://www.mat.puc-rio.br/~nicolau/olimp/obm-l.html
=


Re:[obm-l] Polinomios com coeficientes inteiros

2006-05-11 Por tôpico claudio\.buffara
-- Cabeçalho original ---

De: [EMAIL PROTECTED]
Para: OBM-l (E-mail) obm-l@mat.puc-rio.br
Cópia: 
Data: Thu, 11 May 2006 16:38:26 -0300
Assunto: [obm-l] Polinomios com coeficientes inteiros

 Alguem conhece este teorema?
 Suponhamos que P seja um polinomio do grau n com coeficientes inteiros e
 tenha um numero impar de coeficientes impares, incluindo, dentre estes
 ultimos, os coeficientes do termo independente e do termo dominante. Entao,
 P nao tem raizes a + b*i nas quais a e b sejam ambos racionais. O que
 implica que P nao admite raizes reais racionais.
 Eu vi um esquema da demonstracao, nao entendi tudo. No caso especifico de
 n=2 a demosntracao eh simples.
 Artur
 
 

Suponhamos que (a + bi)/c seja uma raiz de p(x), com a, b e c inteiros, c  0 
e mdc(a,b,c) = 1 (se mdc(a,b,c)  1, 
poderiamos cancelar este fator comum de a, b e c).
Nesse caso, (a - bi)/c tambem eh raiz == 
p(x) eh divisivel por c^2x^2 - 2acx + (a^2+b^2) (em Z[x])

Como o coeficiente lider e o termo independente de p(x) sao impares, temos que 
c^2 e a^2+b^2 sao impares, pois sao fatores 
do coeficiente lider e do termo independente, respectivamente. 

A condicao nos coeficientes significa que se z eh um inteiro impar, entao p(z) 
tambem eh impar.
Em particular p(1) eh impar.

p(1) = c^2 - 2ac + (a^2+b^2) = impar - par + impar = par == contradicao

Logo, p(x) nao admite raizes em Q(i).

Acho que eh isso.

[]s,
Claudio.



=
Instruções para entrar na lista, sair da lista e usar a lista em
http://www.mat.puc-rio.br/~nicolau/olimp/obm-l.html
=


[obm-l] Re:[obm-l] problema do almoço

2006-05-10 Por tôpico claudio\.buffara
Um amigo chega em t = X e sai em t = X+10, onde 0 = X = 60
O outro chega em t = Y e sai em t = Y+10, onde 0 = Y = 60.
Naturalmente, eles se encontram se e somente se um chega antes do outro sair, 
ou seja, se e somente se:
X = Y+10  e  Y = X+10  ==  X-10 = Y = X+10

Fazendo um grafico, vemos que o espaco amostral dos pontos (X,Y) eh o quadrado 
[0,60]x[0,60], cuja area eh igual a 60^2.
Os pontos (X,Y) que interessam estao entre as retas Y = X-10 e Y = X+10.
Logo, a regiao de interesse eh em hexagono cujos vertices sao:
(0,0), (10,0), (60,50), (60,60), (50,60) e (0,10) e cuja area eh igual a 60^2 - 
2*(1/2)*50^2 = 60^2 - 50^2.

Assim, a probabilidade de um encontro eh igual a (60^2 - 50^2)/60^2 = 1 - 25/36 
= 11/36.
 
[]s,
Claudio.

-- Cabeçalho original ---

De: [EMAIL PROTECTED]
Para: obm-l@mat.puc-rio.br
Cópia: 
Data: Tue, 9 May 2006 07:05:48 -0700 (PDT)
Assunto: [obm-l] problema do almoço

 Srs, peço ajuda na resolução deste problema:
 
 
 
 Dois amigos combinaram um encontro para almoçar entre
 12:00 e 13:00h. Alguns dias depois, ambos esquecem o
 momeno exato, mas nenhum deles desiste de ir ao
 encontro, e ambos resolvem ir ao encontro escolhendo a
 hora de chegar aleatoreamente (e independentemente)
 entre 12:00 e 13:00h. Se cada um deles desiste
 esperar, no máximo, 10 min, qual a probabilidade dos
 dois amigos almoçarem juntos neste dia?(eles não tem
 celular!)
 
 
 __
 Do You Yahoo!?
 Tired of spam?  Yahoo! Mail has the best spam protection around 
 http://mail.yahoo.com 
 =
 Instruções para entrar na lista, sair da lista e usar a lista em
 http://www.mat.puc-rio.br/~nicolau/olimp/obm-l.html
 =
 
 


=
Instruções para entrar na lista, sair da lista e usar a lista em
http://www.mat.puc-rio.br/~nicolau/olimp/obm-l.html
=


Re: [obm-l] Ajuda

2006-05-10 Por tôpico claudio\.buffara
Eu acho que este argumento é falho pois ao dividir (2m)!*(2n)! por m!*n! você pode "perder" os fatores primos que fariam com que o quociente fosse divisível por (m+n)!.

Um jeito de resolveré provando que cada primo aparece em (2m)!*(2n)! com um expoente igual ou maior do queo expoentecom que este primo aparece em m!*n!*(m+n)!.

Ou seja, temos que provar que, para cada primo p,
SOMA(k=1) ([2m/p^k] + [2n/p^k]) = 
SOMA(k=1) ([m/p^k] + [n/p^k] + [(m+n)/p^k]

([x] = maior inteiro menor ou igual a x)

Para isso é suficiente provar a desigualdade:
[2x] + [2y] = [x] + [y] + [x+y], onde x e y são reais quaisquer.

x = [x] + {x}
y = [y] + {y} ==

[2x] + [2y] = 2[x] + 2[y] + [2{x}] + [2{y}] (i)
[x] + [y] + [x+y] = 2[x] + 2[y] + [{x}+{y}] (ii)

Subtraindo (ii) de (i), obtemos:
[2x] + [2y] -[x]- [y]- [x+y] = [2{x}] + [2{y}] - [{x}+{y}]

Assim, o problema se reduz a provar que, se a e b pertencem a [0,1), então
[2a] + [2b]= [a+b].

Supondo s.p.d.g. que 0 = a = b  1, vamos por casos:
i) 0 = a =b  1/2 == 0 = 0
ii) 0 = a  1/2 = b  1 == 1 = 0 ou 1
ii) 1/2 = a = b  1 == 2 = 1 

Logo, a desigualdade vale sempre e acabou...

***

O mais legal, entretanto, é achar algum problema de combinatória onde um dado conjunto tenha (2m)!*(2n)!/(m!*n!*(m+n)!) elementos.


[]s,
Claudio.





De:
[EMAIL PROTECTED]




Para:
obm-l@mat.puc-rio.br




Cópia:





Data:
Tue, 9 May 2006 20:48:46 -0300




Assunto:
Re: [obm-l] Ajuda
 m! e n! esta contido em 2m! e 2n!, falta so provar que (m+n)! esta contido em 2m ou 2n fatorial desenvolvidos.
 caso em que m=n
 m+n0=2m=2n
 o que da resultado inteiro
 m maior que n ou n maior que m
 2m ou 2n maior que m+n, o qque demonstra que o denominador tambem se anula neste caso, como m e n sao inteiros, o numerador vais ser uma produto de numeros inteiros.
 
 On 5/9/06, Manoel P G Neto Neto [EMAIL PROTECTED] wrote:

Olá amigos da lista,Vocês poderiam me ajudar com a questão:Sejam m, n inteiros positivos, então(2m)! (2n)! / m! n! (m+n)!é um número inteiro.Grato. 



Abra sua conta no Yahoo! Mail - 1GB de espaço, alertas de e-mail no celular e anti-spam realmente eficaz.




[obm-l] 4 esferas e um tetraedro

2006-05-10 Por tôpico claudio\.buffara

O conjunto A consiste de 4 esferas de raio R cujos centros coincidem com os 
vertices de um tetraedro regular de aresta 2R.
Qual a aresta do menor tetraedro regular que pode ser circunscrito a A?

[]s,
Claudio.



=
Instruções para entrar na lista, sair da lista e usar a lista em
http://www.mat.puc-rio.br/~nicolau/olimp/obm-l.html
=


Re:RES: [obm-l] Somatorios de potencias dos naturais

2006-05-09 Por tôpico claudio\.buffara
Eu ainda prefiro uma demonstração combinatória.

Problema: Quantos ternos ordenados (x,y,z) existem cujos elementos pertencem a {1, 2, 3, ..., n, n+1} e são tais que x  y e x  z?

Solução 1:
Para x = k+1 (k em {1, 2, ..., n}), temos k escolhas para y e k escolhas para z. Logo, existem k^2 ternos da forma (k+1,y,z) nas condições do enunciado.
Fazendo k variar de 1 a n, obtemos que o número total de ternos é:
1^2 + 2^2 + 3^2 + ... + n^2.

Solução 2:
Os ternos (x,y,z) com x  y e x  z são de três tipos:
1. Ternos em que x  y  z
2. Ternos em que x  z  y
3. Ternos em que x  y = z.
Existem Binom(n+1,3) ternos dos tipos 1 e 2 e Binom(n+1,2) ternos do tipo 3.
Logo, o número total de ternos é 2*Binom(n+1,3) + Binom(n+1,2) =
2*(n+1)*n*(n-1)/6 + (n+1)*n/2 =
n*(n+1)*((n-1)/3 + 1/2) =
n*(n+1)*(2n-2+3)/6 =
n*(n+1)*(2n+1)/6

Como ambas as soluções têm que dar o mesmo resultado...

***

Pra soma dos cubos, teríamos que considerar as quádruplas ordenadas (x,y,z,w) de elementos de {1,2,...,n,n+1} tais que x  y, x  z e x  w.

Na solução 2, os tipos básicos de quádrupla seriam:
1. x  y  z  w (total de 6 permutações de y, z e w)
Contribuição = 6*Binom(n+1,4) 
2. x  y = z  w (total de 3)
Contribuição = 3*Binom(n+1,3)
3. x  y  z = w (total de 3)
Contribuição = 3*Binom(n+1,3)
4. x  y = z = w (total de 1)
Contribuição = Binom(n+1,2)

Total = 6*(Binom(n+1,4) + Binom(n+1,3)) + Binom(n+1,2) =
6*Binom(n+2,4) + Binom(n+1,2) =
6*(n+2)*(n+1)*n*(n-1)/24 + (n+1)*n/2 =
(n+1)*n/2 * ((n+2)*(n-1)/2 + 1) =
n*(n+1)/2 * (n^2 + n - 2 + 2)/2 =
n*(n+1)/2 * n*(n+1)/2 =
n^2*(n+1)^2/4

No entanto, será que o fato de ser:
1^3 + 2^3 + ... + n^3 = (1 + 2 + ... + n)^2 
não dá margem a alguma demonstração geométrica?

[]s,
Claudio.





De:
[EMAIL PROTECTED]




Para:
obm-l@mat.puc-rio.br




Cópia:





Data:
Mon, 8 May 2006 16:01:17 -0300




Assunto:
RES: [obm-l] Somatorios de potencias dos naturais
 Vou olhar o seu blog assim que tiver tempo para uma avaliação cuidadosa.
 
 Uma forma de se chegar aa formula para as potências p+1, p inteiro, dos n
 primeiros inteiros positivos eh usar recorrecia. Sendo Bin(p,k) =
 p!/(k!*(n-k)*), k=0, 1,... p, temos pelo Binomio de Newton, temos:
 
 (n + 1)^p = n^p + p*n^(p-1) +Bin(p,k)n^k ...+ 1 
 (n-1 +1)^p = (n-1)^p + p*(n-1)^(p-1) + Bin(p,k)(n-1)^k ...+ 1 
 .
 .
 (1+ 1)^p n = 1 + p+ Bin(p,k)..+1
 
 Somando-se estas n igualdades e fazendo algumas transformacoes algebricas um
 tanto bracais, obtemos a soma das potencias p conhecendo-se a formula das
 potencias de ordem p-1,...1. Isto vai nos mostrar que a soma das potencias p
 eh dada por um polinomio em n do grau p+1.
 Com um pouco de paciencia e muita atencao para nao errar, podemos
 generalizar este processo para obter a formula da soma das potencias de
 ordem p dos n primeiros termos de uma progressao aritmetica. 
 
 Uma vez provado que a soma desejada eh um polinomio de grau p+1 em n ( que
 pode ser feito por inducao em p), vc tambem pode chegar aos coeficientes do
 polinomio atribuindo p+1 valores a n e resolvendo um sistema de equacoes
 lineares. Tambem exige uma certa dose de paciencia. 
 
 De forma simples, eh possivel demonstrar por inducao que S(n,3) = (S(n,1))^2
 = (n*(n+1)/2)^2
 
 Artur
 
 
 
 
 -Mensagem original-
 De: [EMAIL PROTECTED] [mailto:[EMAIL PROTECTED]
 nome de Bruno Bonagura
 Enviada em: segunda-feira, 8 de maio de 2006 14:33
 Para: obm-l@mat.puc-rio.br
 Assunto: [obm-l] Somatorios de potencias dos naturais
 
 
 Olá pessoal,
 
 Na primeira vez em que vi o somatório 1² + 2² + 3² + ... + n² e sua 
 fórmula (1/6)(2n+1)(n+1)n fiquei curioso em tentar demonstrar tal 
 fórmula. Isso foi há quase dois anos! Desde então pensava frequentemente 
 no assunto e as vezes procurava sobre ele na internet. Visito alguns 
 fóruns de matemática, tanto nacionais como internacionais, e sempre que 
 era questionada demonstração para tal fórmula mostravam aquela que 
 utiliza combinação e mais algumas coisas. Confesso que não dei muita 
 atenção para tal demonstração, não tive simpatia com ela.
 
 Enfim, depois de algumas idéias e algumas observações dos azuleijos do 
 banheiro (rs), criei uma demonstração para tal fórmula. Não sei se já 
 foi feita, mas estou sendo sincero ao dizer que a criei sem consultar 
 algo semelhante já produzido. Gostaria que olhassem, criticassem 
 possíveis erros, etc. Mas, principalmente, me digam se já está na 
 literatura corrente esta demonstração. Ela está disponível no meu blog 
 (http://bbonagura.blog.uol.com.br/) no post com título "Empilhando 
 quadrados".
 
 Vale ressaltar que não estou enviando essa mensagem para a lista apenas 
 para fazer propaganda e conseguir visitas no meu blog. Meu intuito é 
 compartilhar conhecimento e receber críticas/sugestões, não a coloco 
 diretamente aqui por causa das fórmulas matemáticas e imagens que a 
 envolvem.
 
 Bruno Bonagura
 


[obm-l] Re:[obm-l] Divisão de polinôm ios

2006-05-05 Por tôpico claudio\.buffara
-- Cabeçalho original ---

De: [EMAIL PROTECTED]
Para: obm-l@mat.puc-rio.br
Cópia: 
Data: Thu, 4 May 2006 18:52:52 -0300
Assunto: [obm-l] Divisão de polinômios

 Olá à todos da lista, esse é o primeiro tópico que inicio aqui.  Estudando
 divisibilidade de polinômios me deparei com o seguinte exercício (a fonte
 diz que é IME, mas não encontrei esse exercício entre os exercícios do IME):
 
 Prove que o polinômio p(x) = x^ + x^ + x^ + ... + x^ + 1 é
 divisível por g(x)= x^9 + x^8 + x^7 +  + x^1 + 1
 
 Creio eu que tenha que utilizar a teoria das congruências (mod). agradeço
 desde já pela ajuda.
 

Basta observar que as raizes de g(x) sao justamente as raizes decimas da 
unidade distintas de 1 e que, se w eh uma tal raiz, entao 
w = w^, pois w^1110 = (w^10)^111 = 1^111 = 1.
Assim, para toda raiz decima da unidade distinta de 1, p(w) = g(w), ou seja, 
g(x) divide p(x).

[]s,
Claudio.




=
Instruções para entrar na lista, sair da lista e usar a lista em
http://www.mat.puc-rio.br/~nicolau/olimp/obm-l.html
=


Re:[obm-l] multiplo de 4....

2006-05-05 Por tôpico claudio\.buffara
-- Cabeçalho original ---

De: [EMAIL PROTECTED]
Para: obm-l@mat.puc-rio.br
Cópia: 
Data: Fri, 5 May 2006 01:20:28 -0300
Assunto: [obm-l] multiplo de 4

 V se alguem pode me ajudar com ajuda com essa
 
 
 Sejam n, a1, a2, a3,...,an, números inteiros tais que a1.a2.a3.an=n e 
 a1+a2+ a3+...+an=0. Prove que n é múltiplo de 4
 

Suponhamos que n nao seja multiplo de 4.

Caso 1: n eh par == n = 4k+2
Nesse caso, apenas um dos a_i (digamos, a_n - spdg) serah par (de fato, serah o 
dobro de um impar).
Teremos entao, a_1 + ... + a_(n-1) = -a_n.
Mas o lado esquerdo eh uma soma de um numero impar de parcelas impares, logo eh 
impar, e o lado direito eh par == 
contradicao.

Caso 2: n eh impar
Nesse caso, todos os a_i serao impares e, portanto, sua soma deverah ser impar 
(soma de um numero impar de parcelas 
impares), ou seja, nunca poderah ser zero == contradicao.

Logo, a unica alternativa que resta eh a de que n eh multiplo de 4.

Este caso pode realmente ocorrer. 
Por exemplo, n = 4, a_1 = 1, a_2 = -1, a_3 = 2 e a_4 = -2.

[]s,
Claudio.



=
Instruções para entrar na lista, sair da lista e usar a lista em
http://www.mat.puc-rio.br/~nicolau/olimp/obm-l.html
=


Re:[obm-l] racionais e inteiros...

2006-05-05 Por tôpico claudio\.buffara

 Ache todos as racionais a tais que 1/4= a =3/4 e que (4a-1)/(27a^4) seja 
 inteiro.
 

Seja a = m/n, com m e n inteiros positivos primos entre si.

1/4 = m/n = 3/4   e   (4m/n - 1)/(27m^4/n^4) = k = inteiro nao-negativo, pois 
a = 1/4 ==

n^3(4m - n) = 27km^4

m | 27km^4 == m | n^3(4m - n) == m | 4m - n == m | n == m = 1 ==

n^3(4 - n) = 27k = 0 
e
1/4 = 1/n = 3/4 == 4/3 = n = 4

== n pertence a {2, 3, 4}

Testando estes valores, vemos que n = 3 e n = 4 satisfazem, correspondendo a k 
= 1 e k = 0, respectivamente.

Nesse caso, as solucoes sao a = 1/4 e a = 1/3.


[]s,
Claudio.






=
Instruções para entrar na lista, sair da lista e usar a lista em
http://www.mat.puc-rio.br/~nicolau/olimp/obm-l.html
=


Re:[obm-l] reais somando 1/3

2006-05-05 Por tôpico claudio\.buffara





De:
[EMAIL PROTECTED]




Para:
obm-l@mat.puc-rio.br




Cópia:





Data:
Fri, 5 May 2006 08:56:14 -0300




Assunto:
[obm-l] reais somando 1/3
 Ache todos os numeros reais x e y tais que 
 
 (1-x)^2 + (x-y)^2 +y^2 = 1/3

1 - 2x + x^2 + x^2 - 2xy + y^2 + y^2 - 1/3 = 0

x^2 - xy + y^2 - x + 1/3 = 0

x^2 - xy + y^2/4 + 3y^2/4 - x + 1/3 = 0

(x - y/2)^2 + 3(y/2)^2 - x + 1/3 = 0

(x - y/2)^2 + 3(y/2)^2 - (x - y/2) - y/2+ 1/3 = 0

Fazendo u = x - y/2 e v = y/2, teremos:

u^2 + 3v^2 - u - v + 1/3 = 0

u^2 - u + 1/4 + 3(v^2 - v/3 + 1/36) + 1/3 - 1/4 - 1/9 = 0

(u - 1/2)^2 + 3(v - 1/6)^2 = 1/36 

Isso é a equação de uma elipse.

[]s,
Claudio.



[obm-l] Resultante de Vetores

2006-05-05 Por tôpico claudio\.buffara
Aqui vai um interessante:

Seja ABC um triângulo cujo circuncentro é O.
Qual a resultante (soma) dos vetores OA, OB e OC?

[]s,
Claudio.



Re:RES: [obm-l] Soma dos quadrados dos divisores

2006-05-03 Por tôpico claudio\.buffara
Dada a fatoração em primos de um inteiro, é fácil obter a soma dos quadrados dos seus divisores. Também é fato que n e n+1 não tem nenhum fator primo em comum. Mas daí a uma solução analítica acho que vai uma boa distância.

O problema está no capítulo 1 do livro "Funções Aritméticas - Números Notáveis" do Edgard de Alencar Filho.

[]s,
Claudio.





De:
[EMAIL PROTECTED]




Para:
obm-l@mat.puc-rio.br




Cópia:





Data:
Wed, 3 May 2006 11:04:31 -0300




Assunto:
RES: [obm-l] Soma dos quadrados dos divisores
 Serah que eh possivel resolver isto analiticamente?
 Artur

-Mensagem original-De: [EMAIL PROTECTED] [mailto:[EMAIL PROTECTED]Em nome de claudio.buffaraEnviada em: terça-feira, 2 de maio de 2006 19:14Para: obm-lAssunto: [obm-l] Soma dos quadrados dos divisores
 Aqui vai um que está dando trabalho:
 
 Ache todos os pares de inteiros positivos consecutivos cujas respectivas somas dos quadrados dos divisores positivos são iguais.
 
 Por inspeção, eu achei 6 e 7 (1^2 +2^2 +3^2 + 6^2 = 1^2 + 7^2) mas não consegui achar outras nem provar que esta é a única solução.
 
 []s,
 Claudio.
 


[obm-l] Re:[obm-l] Condição (mais geral) para diferenciabilidade de uma funç ão

2006-05-02 Por tôpico claudio\.buffara
Veja o livro Curso de Análise - vol. 2 do Elon Lages Lima, em particular a observação que se segue ao Teorema 1 do cap. III - seção 3.

[]s,
Claudio.





De:
[EMAIL PROTECTED]




Para:
"obm-l" obm-l@mat.puc-rio.br




Cópia:





Data:
Tue, 2 May 2006 08:58:45 -0300




Assunto:
[obm-l] Condição (mais geral) para diferenciabilidade de uma função

 Encontrei esta questão em um outro forum:
 http://at.yorku.ca/cgi-bin/bbqa?forum=ask_an_analyst;task=show_msg;msg=0851
 
 Achei interessante mas não consegui resolver até agora. Alguém poderia me dar alguma luz. 
 
 Abaixo reescrevo a questão (que aparentemente foi retirada do Spivak - Calculus on Manifolds).
 
 Seja f: R^{2} - R.
 
 As derivadas parciais existem em uma vizinhança do ponto (a,b).
 APENAS uma das derivadas parciais é continua em (a,b).
 Então 
 f é diferenciável em (a,b).
 
 Em todos os livros que estudei, lembro apenas de ter visto este resultado com a hipótese de que TODAS as derivadas parciais eram continuas no ponto de interesse.
 
 []'s


Re:[obm-l] PROBLEMA GEO

2006-05-02 Por tôpico claudio\.buffara
arcsen(1 - 2b^2/a^2), onde a = hipotenusa e b = cateto menor





De:
[EMAIL PROTECTED]




Para:
obm-l@mat.puc-rio.br




Cópia:





Data:
Tue, 02 May 2006 14:51:41 -0300




Assunto:
[obm-l] PROBLEMA GEO
 
 Srs
 
 
 Qual é o ângulo formado pela mediana e a altura referentes à
 hipotenusa?
 
 
 
 
 obrigado
 
 
 Sarmento
 
 Aqui na Oi Internet você ganha ou ganha. Além de acesso grátis com qualidade,
 ganha contas ilimitadas de email com 1 giga cada uma. Ganha 60 mega 
 para hospedar
 sua página pessoal. Ganha flog, suporte grátis e muito mais. Baixe grátis o
 Discador em http://www.oi.com.br/discador e comece a ganhar.
 
 Agora, se o seu negócio é voar na internet sem pagar uma fortuna, assine
 Oi Internet banda larga por apenas R$ 9,90. Clique em
 http://www.oi.com.br/bandalarga e aproveite essa bocada!
 
 
 
 =
 Instruções para entrar na lista, sair da lista e usar a lista em
 http://www.mat.puc-rio.br/~nicolau/olimp/obm-l.html
 =
 


[obm-l] Soma dos quadrados dos divisores

2006-05-02 Por tôpico claudio\.buffara
Aqui vai um que está dando trabalho:

Ache todos os pares de inteiros positivos consecutivos cujas respectivas somas dos quadrados dos divisores positivos são iguais.

Por inspeção, eu achei 6 e 7 (1^2 +2^2 +3^2 + 6^2 = 1^2 + 7^2) mas não consegui achar outras nem provar que esta é a única solução.

[]s,
Claudio.



Re:[obm-l] Desigualdade

2006-04-29 Por tôpico claudio\.buffara
-- Cabeçalho original ---

De: [EMAIL PROTECTED]
Para: obm-l@mat.puc-rio.br
Cópia: 
Data: Sat, 29 Apr 2006 00:45:23 + (GMT)
Assunto: [obm-l] Desigualdade

 Quem puder me ajudar agradeço.

   1/2 * 3/4 * 5/6*...*99/1001/12

 

A = 1/2 * 3/4 * 5/6 * 7/8 * 9/10 * ... * 99/100

B = 1/2 * 3/4 * 5/6 * 8/9 * 10/11 * ... * 100/101

A  B == A^2  AB

AB = (1/2)^2 * (3/4)^2 * (5/6)^2 * 7/8 * 8/9 * ... * 99/100 * 100/101 = 
225/2304 * 7/101 = 1575/232704  1/144

Logo, A^2  AB  1/144 == A  1/12

[]s,
Claudio.
 


=
Instruções para entrar na lista, sair da lista e usar a lista em
http://www.mat.puc-rio.br/~nicolau/olimp/obm-l.html
=


Re:[obm-l] Achar as raizes z^4+4

2006-04-28 Por tôpico claudio\.buffara
z^4 + 4 = 
z^4 + 4z^2 + 4 - 4z^2 =
(z^2 + 2)^2 - (2z)^2 =
(z^2 + 2z + 2)*(z^2 - 2z + 2)





De:
[EMAIL PROTECTED]




Para:
obm-l@mat.puc-rio.br




Cópia:





Data:
Fri, 28 Apr 2006 11:03:49 -0300




Assunto:
[obm-l] Achar as raizes z^4+4 
 
 Favor como achar as raizes
 
 Ache as 4 raizes da equação z^4+4 = 0: Use-as para fatorar z^4+4 em fatores
 quadraticos com coeficientes reais.
 
 
 
 
 =
 Instruções para entrar na lista, sair da lista e usar a lista em
 http://www.mat.puc-rio.br/~nicolau/olimp/obm-l.html
 =
 


[obm-l] Diferença de Quadrados (era: Re: [ obm-l] Algebra)

2006-04-28 Por tôpico claudio\.buffara
Esse problema tem uma generalização interessante:
1. Ache todos osnaturais que podem ser representados como uma diferença de quadrados de naturais;
2. Para quais deles a representação é única?

Por exemplo, se p é um primo ímpar, então:
a^2 - b^2 = p ==
(a + b)(a - b) = p ==
a + b = p e a - b = 1 ==
a = (p+1)/2 e b = (p-1)/2
e essa representação é (claramente?) única.

[]s,
Claudio.





De:
[EMAIL PROTECTED]




Para:
obm-l@mat.puc-rio.br




Cópia:





Data:
Fri, 28 Apr 2006 09:42:50 -0300




Assunto:
Re: [obm-l] Algebra
 Vejamos:
  a^2 - b^2 = 7
  (a+b)(a-b) = 7
 
 Vamos por exclusão:
  a-b não pode ser 0 
  a-b não pode ser 3 (pois 3 não divide 7)
  a-b não pode ser 4 (pois 4 não divide 7)
  a-b não pode ser 7 
  aqui é interessante: se a = 7+b e substituindo acima temos que:
 ( 7+b+b) 7 = 7
  (7+2b) = 1
  2b = -6 == b=-3 que não é natural
 Resposta B.

- Original Message - 
From: Bruna Carvalho 
To: obm-l@mat.puc-rio.br 
Sent: Thursday, April 27, 2006 8:38 PM
Subject: [obm-l] Algebra
 Os números naturais a e b, com ab, são tais que a^2-b^2=7. O valor de a-b é:a)0 b)1 c)3 d)4 e)7 


Re: [obm-l] T. Numeros

2006-03-28 Por tôpico claudio\.buffara
Se os cubos tiverem que ser não-negativos, então isso é falso. 
Tente expressar 23 como soma de cubos.
O mínimo número de cubosnão-negativos necessário pra expressar qualquer inteiro positivo (como uma soma de cubos) é 9 e, se você tiver uma prova por indução desse fato, eu gostaria muito de vê-la.

Por outro lado, se os cubos puderem ser negativos, então: 
23 = 27+ (-1)+ (-1)+ (-1)+ (-1).
No entanto, eu não sei se 5 cubos são sempre suficientes.

Procure "Waring's problem" no Google.

[]s,
Claudio.





De:
[EMAIL PROTECTED]




Para:
obm-l@mat.puc-rio.br




Cópia:





Data:
Tue, 28 Mar 2006 15:15:10 -0300




Assunto:
Re: [obm-l] T. Numeros
 Todo inteiro, ou todo inteiro maior que 5?
 Para todos os inteiros menores que 5 basta tomar os
 primeiros cubos iguais a zero:
 
 1 = 0^3 + 0^3 + 0^3 + 0^3 + 1^3, etc ...
 
 Para inteiros maiores que 5, deve haver algum truque que 
 permita concluir quese n se escreve como soma de cubos
 então n+1 também se escreve como a soma de cubos.
 Daía a prova sai por indução.
 

- Original Message - 
From: Klaus Ferraz 
To: obm-l@mat.puc-rio.br 
Sent: Friday, March 17, 2006 11:57 PM
Subject: [obm-l] T. Numeros
 
Mostre que todo inteiro pode ser escrito como soma de 5 cubos.



Re:[obm-l] P.A

2006-03-14 Por tôpico claudio\.buffara
Seja p um termo dessa progressão, cuja razão é r.
S.p.d.g. podemos supor que r é um inteiro positivo.
p é obviamente primo, senão acabou.
Mas então, p + p*r = p*(1 + r) é um termo da progressão e é composto.
Logo, uma tal progressão não pode existir.

Seja S = SOMA(k=1...100) x_k.
x_k = S - x_k - k== 2*x_k = S - k.
Somando com k variando de 1 a 100, obtemos:
2*S = 100*S - 5050 == S = 2525/49.
x_50= (2525/49 - 50)/2 = 75/98.

[]s,
Claudio.





De:
[EMAIL PROTECTED]




Para:
obm-l@mat.puc-rio.br




Cópia:





Data:
Tue, 14 Mar 2006 01:11:41 + (GMT)




Assunto:
[obm-l] P.A
 Existe uma progressao aritmetica infinita de razao diferente de zero que pode ser formada apenas por numeros primos ? Prove
 
 A sequencia X_1,X_2,...,X_100 é tal que cada x_k é igual a k a menos que a soma dos outros 99 numeros. Determine x_50.
 75/98



[obm-l] Inteiros da forma ax + by

2006-03-09 Por tôpico claudio\.buffara
Dados inteiros positivos a, b com mdc(a,b) = 1, o problema é encontrar todos os inteiros positivos que podem ser representados na forma ax + by, onde x e y são inteiros não-negativos.

Nesse caso:
1) ab - a - b não pode ser representado;
2) todo inteiro maior do queab - a - b pode ser representado;
3) exatamente metade dos inteiros no intervalo [0,ab - a - b] pode ser representada.

Tudo isso está provado em:
http://www.cut-the-knot.org/blue/Byzantine.shtml

[]s,
Claudio.



[obm-l] Re: [obm-l] Re: [obm-l] O que é ma is fácil????

2006-02-16 Por tôpico claudio\.buffara
Oi, Denisson:

Os links não tem nenhum conteúdo matemático relevante. 
Só valem a pena por causa da indignação demonstrada pelo tal Fabiano Sutter (o suposto quebrador do RSA) com o ceticismo de outros participantes da lista. Até hoje não sei se ele estava falando sério ou não, mas como ele sumiu, é bem provável que estivesse.

O thread começa em:
http://www.mat.puc-rio.br/~nicolau/olimp/obm-l.200404/msg00296.html

[]s,
Claudio.





De:
[EMAIL PROTECTED]




Para:
obm-l@mat.puc-rio.br




Cópia:





Data:
Thu, 16 Feb 2006 00:30:54 -0300




Assunto:
Re: [obm-l] Re: [obm-l] O que é mais fácilBuffara, fiquei curioso, sabe dizer o link dessas msgs que foram enviadas sobre o kara supostamente ter encontrado tal algoritmo?SdsDenisson
 Em 15/02/06, claudio.buffara [EMAIL PROTECTED] escreveu:

 




De:
[EMAIL PROTECTED]




Para:
obm-l@mat.puc-rio.br




Cópia:





Data:
Wed, 15 Feb 2006 12:30:37 -0200




Assunto:
Re: [obm-l] O que é mais fácil
  
  Eh um problema em aberto decidir se existe um algoritmo de tempo polinomial
  para fatorar inteiros. O certo eh que nenhum tal algoritmo eh conhecido
  ate hoje. Parece seguro apostar, entretanto, que mesmo se tal algoritmo
  existir, fatorar sempre serah mais dificil do que testar primalidade.
  
  
  []s, N.
  
 
 Mas tem um cara que andou frequentando a lista obm-l e insinuando que estava perto de encontrar um algoritmo para fatorar inteiros (e com isso quebrar o RSA, como ele mesmo costumava dizer). Como ele anda sumido, de duas uma: ou ele finalmente conseguiu e foi prontamente sequestrado pela CIA, ou continua emburrado ese sentindo ofendido pelas mensagens céticas que recebeu... 
 
 []s,
 Claudio.
 
 -- Denisson"Você nasce sem pedir mas morre sem querer.Aproveite esse intervalo!"


Re: [obm-l] Uma Curva Interessante

2006-02-08 Por tôpico claudio\.buffara
Acho queo argumento do Wagner se aplica desde que L  distancia entre os dois pontos, pois o segmento quase horizontal pode ser arbitrariamente curto.

[]s,
Claudio.





De:
[EMAIL PROTECTED]




Para:
obm-l@mat.puc-rio.br




Cópia:





Data:
Tue, 7 Feb 2006 21:39:15 -0300




Assunto:
Re: [obm-l] Uma Curva InteressanteCaro Wagner, ainda não tentei resolver o problema, mas tenho a impressão que ele pode estar "bem posto", sim, pois a curva deve ter comprimento fixo L. Parece-me que esse fato faz com que seu argumento não se aplique, não? Se eu estiver correto, parece interessante essa "braquistócrona inversa".Leonardo Maia
 On 2/7/06, Eduardo Wagner [EMAIL PROTECTED] wrote:
Oi Paulo:Oi Pessoal:Esta eh facil. Faça um primeiro trecho quase, maisquase mesmo, horizontal e o restante do comprimentocaindo "abruptamente" até B. Como nesse primeiro trechoa inclinação pode ser tão pequena quanto se queira, o tempo que a bolinha demora para percorre-lo pode serarbitrariamente grande.Logo, não existe a curva do tempo máximo.Abraços,Wagner.--From: "Paulo Santa Rita"  [EMAIL PROTECTED]To: obm-l@mat.puc-rio.brSubject: [obm-l] Uma Curva InteressanteDate: Tue, Feb 7, 2006, 2:14 PM  Ola Pessoal, Alguem me propos o seguinte problema : "Dentre todas as curvas de mesmo comprimento L que ligam dois pontos A e B de um plano, determinar aquela em que um corpo submetido exclusivamente ao  campo gravitacional da terra (suposto constante ) gasta o tempo maximo para ir de A para B." NOTA : Se A=(Xa,Ya) e B=(Xb,Yb) sao as coordenadas de A e B suponha que Xb  Xa e  Yb  Ya. Tambem suponha que : distancia entre A e BL (Xb - Xa) + (Ya - Yb) Parece ser um problema interessante, nao trivial. Como estou sem tempo pra pensar nele, estou passando pra voces.  Um Abraco a Todos Paulo Santa Rita 3,1414,070206 _ Facilite sua vida: Use o Windows Desktop Search e encontre qualquer arquivo  ou e-mail em seu PC. Acesse:http://desktop.msn.com.br = Instruções para entrar na lista, sair da lista e usar a lista em  http://www.mat.puc-rio.br/~nicolau/olimp/obm-l.html == Instruções para entrar na lista, sair da lista e usar a lista emhttp://www.mat.puc-rio.br/~nicolau/olimp/obm-l.html= 


Re: [obm-l] Bertrand Russel

2005-12-27 Por tôpico claudio\.buffara
Caros colegas da lista:

Antes de mais nada, espero que, para todos nos,2006 seja muito melhor que 2005 e muito pior que 2007.

No mais, eu lembro de ter lido no livro Curso de Analise - vol. 1 do Elon uma opiniao (se nao me enganoatribuida aSpivak) sobre o conceito de numero, que eh a seguinte:

Nao importa o que sejam os numeros. Isso seria mais uma questao filosofica (e, portanto, fora do escopo da matematica). O que importa eh como eles se comportam.

Essa atitude me parece satisfatoria, ateh porque a definicao:
"Número é a classe de todos os conjuntos similares a um conjunto dado"
nao significa muita coisa pra mim (por exemplo, o que sao conjuntos similares?). 

Por exemplo, ao inves de me envolver em especulacos metafisicas sobre o que eh o conjunto dos numeros naturais, ou o que eh o numero 1, eu prefiro aceitar sem discutira existencia de um conjunto N, cujos elementos sao chamados "numeros naturais", os quais obedecem aos axiomas de Peano.

Isso me livra de ter que estudar o calhamaco (para mim incompreensivel) de Bertrand Russel e Alfred North Whitehead sobre os fundamentos da matematica e me permite mergulhar direto na parte interessante dessa disciplina - algebra, analise, geometria, topologia, etc. 

Mas, eh claro, isso eh soh a opiniao de um amador...

[]s a todos,
Claudio.






De:
[EMAIL PROTECTED]




Para:
obm-l@mat.puc-rio.br




Cópia:





Data:
Sun, 25 Dec 2005 01:19:29 -0200




Assunto:
Re: [obm-l] Bertrand Russel
 On Wed, Dec 21, 2005 at 11:36:04PM -0300, Denisson wrote:
  Estou lendo o livro História do Pensamento Ocidental de Bertrand Russel e na
  pg 408 ele define o seguinte:
  "Número é a classe de todas as classes similares a uma classe dada"
  Alguém poderia discutir se essa definição é realmente consistente? Não
  fiquei muito seguro com ela. Além disso o que ele estaria querendo dizer com
  'similares'?
 
 Antes de mais nada: esta definição não é muito boa sob o ponto de vista
 de consistência, como você diz. Seria bem melhor se fosse:
 "Número é a classe de todas os conjuntos similares a um conjunto dado"
 
 Isto é uma definição aceitável de número cardinal em uma versão da
 teoria de conjunto que inclua classes. Nesta frase, dois conjuntos
 são similares se existir uma bijeção entre eles.
 Note que esta *não* é a definição de cardinal infinito que você
 encontra na maioria dos livros de teoria dos conjuntos:
 a definição usual é que um cardinal é um ordinal que não é similar
 (no sentido acima) a nenhum de seus elementos, e um ordinal é
 um conjunto transitivo e bem-ordenado pela relação "pertence".
 
 Aliás, acho que agora eu sei de onde os elaboradores do dicionário
 do Aurélio tiraram a definição de número que está lá:
 "Número: conjunto de todos os conjuntos equivalentes a um conjunto dado"
 A troca da palavra "classe" pela palavra "conjunto" é desastrosa:
 em nenhuma das versões usuais da teoria dos conjuntos faz sentido,
 por exemplo, tomar o conjunto de todos os conjuntos unitários.
 Usar isto como a primeira definição de número também é criticável
 sob vários outros pontos de vista, entre eles a total inadequação
 desta definição, mesmo que corrigida, para 99% do público.
 
 Uma curiosidade minha: quando foi que Bertrand Russel escreveu este livro?
 
 []s, N.
 =
 Instruções para entrar na lista, sair da lista e usar a lista em
 http://www.mat.puc-rio.br/~nicolau/olimp/obm-l.html
 =
 


Re: [obm-l] Ainda sobre Teoria dos Números

2005-12-14 Por tôpico Claudio Buffara
on 14.12.05 15:08, Leonardo de Almeida Matos Moraes at [EMAIL PROTECTED]
wrote:

 Pessoal,
 
 uma deficiencia que sempre tive foi em Teoria dos numeros. Como acho que
 nunca e' tarde para aprender, sera' que voces poderiam me indicar uma boa
 bibliografia neste tema?
 
 Abracos desde ja',
 
 Leonardo.

 
De uma olhada em:
http://www.numbertheory.org/ntw/lecture_notes.html
ou entao:
http://www.rzuser.uni-heidelberg.de/~hb3/notes.html

[]s,
Claudio.

=
Instruções para entrar na lista, sair da lista e usar a lista em
http://www.mat.puc-rio.br/~nicolau/olimp/obm-l.html
=


[obm-l] FUTEBOL

2005-11-23 Por tôpico claudio\.buffara
De todos estes problemas futebolísticos que apareceram recentemente na lista, ainda acho o mais interessante este aqui:

Num campeonato com n times, cada par de times joga uma única vez, cada vitória vale 2 pontos, cada empate 1 e cada derrota 0. É fácil ver que o número total de pontos disputados é n(n-1). O problema é determinar uma condição necessária e suficiente para que uma sequência não-crescente de n inteiros não-negativos (a_1, a_2, , a_n) com a_1 + a_2 + ... + a_n = n(n-1) represente a pontuação dos n times ao fim do campeonato.

E se cada vitória valer 3 pontos (que é a regra geral hoje em dia)? 
Nesse caso, não se sabe a priori o número total de pontos marcados, que será igual a 3n(n-1)/2 - E, onde E = no. de jogos que terminaram empatados.

[]s,
Claudio.


[obm-l] Re:[obm-l] questão sobre funçõ es da UFOP fica dando loop :=P

2005-11-22 Por tôpico claudio\.buffara
x = 2 == 2f(2) - f(1/2) = 4
x = 1/2 == -f(2) + 2f(1/2) = 1/4

Resolvendo esse sisteminha pra f(2) e f(1/2) obtemos:
f(1/2) = 3/2 e f(2) = 11/4 ==

2f(2) + f(1/2) = 11/2 + 3/2 =7.

[]s,
Claudio.





De:
[EMAIL PROTECTED]




Para:
obm-l@mat.puc-rio.br




Cópia:





Data:
Tue, 22 Nov 2005 09:26:01 -0300




Assunto:
[obm-l] questão sobre funções da UFOP fica dando loop :=P
 Pessoal, é uma questão bem simples, mas não estou conseguindo desenvolver
 quando chego na parte final do exercício e a funcão fica dando voltas e acaba
 voltando no mesmo lugar, vejam:
 
 (ufop-mg) Seja f:R*-R tal que 2f(x) - f(1/x) = x^2. Então 2f(2) + f(1/2) é
 igual a:
 a)12 b)4 c)1/4 d)17/4 e)7
 
 resp: letra "e"
 
 ps: quando chego em 6,25 + f(1/2)*1/2 , não consigo desenvolver e fica
 voltando em 4,25 + f(2).
 
 obrigado desde já!
 
 =
 Instruções para entrar na lista, sair da lista e usar a lista em
 http://www.mat.puc-rio.br/~nicolau/olimp/obm-l.html
 =
 


[obm-l] Raizes de um Polinomio

2005-11-20 Por tôpico Claudio Buffara
Aqui vai um bonitinho...

Dado o polinomio complexo p(z) = a_0 + a_1*z + ... + a_n*z^n,
prove que se, para algum k em [0,n], vale |a_k|  SOMA(i  k) |a_i|,
entao p(z) tem exatamente k raizes (contadas com multiplicidade) no interior
do disco unitario (|z|  1).

[]s,
Claudio.

=
Instruções para entrar na lista, sair da lista e usar a lista em
http://www.mat.puc-rio.br/~nicolau/olimp/obm-l.html
=


Re: [obm-l] CAMPEONATOS FUTEBOLÍSTICOS!

2005-11-20 Por tôpico Claudio Buffara
Um problema interessante relacionado a esse eh o seguinte:

Dada uma sequencia nao-crescente de 8 inteiros nao-negativos cuja soma eh
56, determinar se esta sequencia pode ou nao representar as pontuacoes dos 8
times ao final do campeonato.

Por exemplo, se o primeiro termo de uma sequencia valida eh 14, entao o
segundo termo tem que ser = 12. Por outro lado, uma sequencia valida pode
ter os dois primeiros termos iguais a 13 (mas nesse caso, o terceiro termo
deve ser = 10).

[]s,
Claudio.


on 20.11.05 11:03, Chicao Valadares at [EMAIL PROTECTED] wrote:

 seguindo esse raciocinio e essa forma de campeonato
 temos que a qt de pontos mínima que garanta com 100%
 de certeza que um time estará nas n/2 maiores
 posiçoes(n par, n é o numero de times) será (Cn,2 *
 2)/n + 1 = n. 
 
 
 --- Chicao Valadares [EMAIL PROTECTED]
 escreveu:
 
 
 Oito times disputam a inclusão no quadrangular
 final
 de um campeonato de
 futebol. Sabe-se que cada par de times joga uma só
 vez entre si e que, em
 caso de vitória, o time ganha dois pontos, no caso
 de empate, ganha um ponto
 e, na derrota, não ganha ponto. Qual é o número
 mínimo de pontos que um time
 deve alcançar para garantir a passagem para o
 quadrangular final?
 
 vou na intuiçao(as vezes, intuiçao na matematica nao
 funciona)...
 
 sao combinaçao de 8,2 = 28 jogos e cada time pode
 fazer de 0 ate 14 pontos pq cada time faz 7
 jogoscomo é tudo simetrico, neste caso, vc
 poderá
 pensar assim: são 2 pontos pra cada jogo ou seja,
 são
 no maximo 56 pontos disputados e tambem a soma dos
 pontos de todos os times no final sao 56 pontos. No
 pior caso teremos 56/8 = 7 pontos pra cada equipe ,
 ou
 seja, situaçao de indefiniçao total mas pra garantir
 a
 classificaçao basta vc ter mais um ponto ou seja 8
 pontos que implica que alguem vai ter 6 pontos. Com
 8
 pontos vc estará concerteza no grupo dos 4 que mais
 pontuaram.Acertei???
 
 


=
Instruções para entrar na lista, sair da lista e usar a lista em
http://www.mat.puc-rio.br/~nicolau/olimp/obm-l.html
=


Re: [obm-l] Re: [obm-l] Uma questão de fatorial

2005-11-10 Por tôpico Claudio Buffara
Title: Re: [obm-l] Re: [obm-l] Uma questão de fatorial



Seja f: R - R dada por:
f(x) = x(x + 1)(x + 2)(x + 3)...(x + 2006) - 1.

Entao:
f(x)/2006! = x(1 + x)(1 + x/2)(1 + x/3)...(1 + x/2006) - 1/2006!

Se x  0, entao f(x)/2006!  x - 1/2006!

Eh claro que f(0) = -1.

Alem disso, 1/2006!  0 == 
f(1/2006!)/2006!  1/2006! - 1/2006! = 0 ==
f(1/2006!)  0

Logo, como f eh continua em toda a reta, o TVI implica que existe a entre 0 e 1/2006! tal que f(a) = 0.

[]s,
Claudio.

on 10.11.05 13:00, Murilo RFL at [EMAIL PROTECTED] wrote:

x(x+1)(x+2)(x+3)...(x+2006) = 1
n=2007 termos (0..2006)
 
Desenvolvendo o polinomio
x(x+1)(x+2)(x+3)...(x+2006) = 1 =
x^2007 +(1+2+3+...+2006)x^2006+ ... + (1*2*3*...*2006)x - 1 = 0.
x^2007 +(sum(1..2006))x^2006+ ... + (2006!)x - 1 = 0.
 
seja x0 eh facil ver q [ x^2007 +(sum(1..2006))x^2006+ ... ] = A tal q A  0
logo a equaçao eh:
A + (2006!)x - 1 = 0.
x = 1/(2006!) - A/2006!
 
como A0 = A/2006!0
e logo x, alguma raiz positiva do polinomio, eh menor de q 1/(2006!) 
 
cqd.
 
[]'s 
MuriloRFL
 
 
 
- Original Message - 
From: Robÿe9rio Alves mailto:[EMAIL PROTECTED] 
To: obm-l@mat.puc-rio.br 
Sent: Thursday, November 10, 2005 11:36 AM
Subject: [obm-l] Uma questão de fatorial

x(x+1)(x+2)(x+3)...(x+2006) = 1 Seja menor menor raiz positiva dessa equaçÃo. Prove que ela seja menor que 1/2006!.

Yahoo! Acesso Grátis: Internet rápida e grátis.
Instale o discador agora! http://us.rd.yahoo.com/mail/br/tagline/discador/*http://br.acesso.yahoo.com/ 







Re:RES: [obm-l] desigualdade

2005-11-08 Por tôpico claudio\.buffara
Ou então,

P = (1/2)*(3/4)*(5/6)*...*(99/100)
Q = (2/3)*(4/5)*(6/7)*...*(100/101)

Claramente, P  Q == 
P^2  PQ = 1/101 ==
P  1/raiz(101)  1/raiz(100) = 1/10

Por outro lado,
R = (1/2)*(2/3)*(4/5)*...* (98/99), de modo que:
P  R ==
P^2  PR = (1/2)*(1/100) = 1/200 ==
P  1/raiz(200)  1/raiz(225) = 1/15

[]s,
Claudio.







De:
[EMAIL PROTECTED]




Para:
obm-l@mat.puc-rio.br




Cópia:





Data:
Tue, 8 Nov 2005 17:55:27 -0200




Assunto:
RES: [obm-l] desigualdade
 De modo geral, para todo n=1 temosP_n = 1/2* 3/4 *(2n-1)/(2n) = Produto(i =1,n)(1 - 1/(2n)). Pela desigualdade MA = MG, para n1 temos que (P_n)^(1/n)  (1/n) * Soma (i=1,n)(1 - 1/(2n)) = 1 - (1 + 1/2 +1/n)/(2*n) .Para n1,vale a desigualdade1 + 1/2 +1/n  ln(n+1), de modo que(P_n)^(1/n)  1 - ln(n+1)/(2n). Finalmente, concluimos que, para n 1, P_n  (1 - ln(n+1)/(2n))^n. No caso, temos n=100, o que nos mostra que . (1/2)*(3/4)*(5/6)*...*(99/100)  0,096849, uma estimativa bem mais rigorosa do que a apresentada. Acho que o limite inferior apresentado estah incorreto.
 Quando n-- oo, vemos que (1 - ln(n+1)/(2n))^n --0, logo P_n -- 0. Na terminologia adotada em produtos infinitos, temos que P_n diverge para 0.
 
 
 Artur






 
 -Mensagem original-De: [EMAIL PROTECTED] [mailto:[EMAIL PROTECTED]Em nome de Danilo NascimentoEnviada em: segunda-feira, 7 de novembro de 2005 20:53Para: obm-l@mat.puc-rio.brAssunto: [obm-l] desigualdade

 Prove a desigualdade. 
 1/15(1/2)*(3/4)*(5/6)*...*(99/100)1/10


Yahoo! Acesso Grátis: Internet rápida e grátis.Instale o discador agora!


Re: [obm-l] m^x + x (off-topic)

2005-11-06 Por tôpico Claudio Buffara
on 02.11.05 14:30, Guilherme Augusto at [EMAIL PROTECTED] wrote:
 
 2) como eu resolvo Soma(1, infinito)(1/i^2) sem
 recorrer a cálculo? Onde eu peguei dizia que era
 possível usando apenas propriedades de somatório. (na
 verdade, pedia para provar que a soma é (pi^2)/6 )
 
De uma olhada no problema 233 do livro THE USSR OLYMPIAD PROBLEM BOOK de
Shklarsky, Chentzov e Yaglom - publicado pela Dover.

A solucao lah contida eh um bom exemplo de um caso em que a solucao
elementar eh muito mais dificil de que a solucao usando calculo.

Alias, de nde voce tirou este problema?

[]s,
Claudio.


=
Instruções para entrar na lista, sair da lista e usar a lista em
http://www.mat.puc-rio.br/~nicolau/olimp/obm-l.html
=


Re: RES: [obm-l] Economia na lista obm-l

2005-11-05 Por tôpico Claudio Buffara
Title: Re: RES: [obm-l] Economia na lista obm-l



OK. Entao aqui vai, jah com desculpas pelo meio-off-topic.

Consideremos o caso de uma opcao de compra com 1 ano de prazo, preco de exercicio = K, sobre um ativo que hoje vale S e, daqui a um ano, vai valer:
H, com probabilidade p ou L, com probabilidade 1-p (L  H)

Isso significa que a opcao dah a seu titular o direito de adquirir o ativo pelo preco K dentro de 1 ano - o caso de interesse eh, naturalmente, quando L  K  H. 
Ou seja, se daqui a um ano o ativo valer H, o titular receberah H - K (ele exercerah a opcao, adquirindo o ativo por K e, imediatamente, poderah vender o ativo no mercado, recebendo H - se ele resolver nao ficar com o ativo, ele estrarah correndo um outro risco, o qual nao tem nada a ver com a opcao). Mas se o ativo valer H, ele nao receberah nem pagarah nada. 

Para nao haver arbitragem (ou seja, lucro garantido com risco zero - algo que nao pode acontecer num mercado verdadeiramente eficiente, coisa que nenhum eh de fato!), a seguinte relacao deve ser satisfeita: L  S(1+i)  H, onde i = taxa de juros (suposta constante ao longo do ano). Pergunta pra voce: por que essa relacao deve valer?

Nesse caso, talvez o mais surpreendente eh que o valor da opcao nao depende de p.
O que depende de p, dados H e L, eh justamente o preco a vista S.
Supondo que o mercado eh avesso a risco (o que me parece razoavel), a seguinte relacao deve prevalecer:
S  (H*p + L*(1-p))/(1+i), de modo que a rentabilidade esperada do ativo serah:
(H*p + L*(1-p))/S - 1  i = taxa de juros de uma aplicacao sem risco

No entanto, o mercado, se for eficiente, soh exige premio de risco (ou seja, uma rentabilidade acima da taxa de juros sem risco) de um dado ativo quando este risco nao for diversificavel.

No caso das opcoes, o risco eh totalmente diversificavel, uma vez que eh possivel construir uma carteira de investimentos composta do ativo-objeto da opcao e de um emprestimo, cujo fluxo de caixa eh exatamente igual ao do ativo. Logo, para nao haver arbitragem a carteira deve valer a mesma coisa que a opcao.

Assim, um investidor que vende a opcao e compra esta carteira nao terah risco algum e, portanto, nao deveria ter lucro algum.

O valor da opcao eh facil de calcular:

Na data inicial, o investidor vende a opcao de compra, arrecadando C, toma um emprestimo de B reais a juro i, e compra n unidades do ativo-objeto.
Logo, seu fluxo de caixa eh igual a C + B - n*S

Na data final: 
1) se o ativo valer H, o investidor pagarah H - K ao comprador da opcao, B*(1+i) ao banco, e receberah n*H pelo ativo

2) se o ativo valer L, o investidor nao pagarah nada ao comprador da opcao, pagarah B*(1+i) ao banco e receberah n*L pelo ativo.
(estou supondo que L  K  H)

Se quisermos zerar o fluxo de caixa na data final, teremos que escolher n e B de modo que:
n*H - (H-K) - B*(1+i) = 0
e
n*L - B*(1+i) = 0.

Resolvendo para n e B, obtemos:
n = (H - K)/(H - L) e B = ((H - K)/(H - L))*L/(1+i)

Se o fluxo de caixa no fim for zero em qualquer cenario, entao o fluxo de caixa inicial serah tambem 0, ou seja, dados n e B solucoes do sistema acima, teremos: 
C = n*S - B = ((H - K)/(H - L))*(S - L/(1+i))

[]s,
Claudio.

on 05.11.05 03:29, José Diogo Barbosa at [EMAIL PROTECTED] wrote:

Olá cláudio 

Gostaria de ver a resposta certa desse problema! Se puder manda pra gente. Acho a resposta do artur muito boa também. Do arthur  muito boa também! Se vc puder mostrar onde errei, vou agradecer 

ps: quase nunca me manifesto9 na lista, mas fico acampando e e sei a sua importância.  

Abraços 

 







Re: RES: RES: [obm-l] Economia na lista obm-l

2005-11-05 Por tôpico Claudio Buffara
Title: Re: RES: RES: [obm-l] Economia na lista obm-l



Nao eh acaso, nao! O que voce fez foi essencialmente a mesma coisa que eu.
Se a opcao nao valesse 19,444 haveria uma possibilidade de arbitragem.

[]s,
Claudio.

on 05.11.05 14:04, José Diogo Barbosa at [EMAIL PROTECTED] wrote:

Cláudio 

 

Não sei se por acaso (imagino que não), o seu raciocínio te levou a dar o mesmo preço que eu dei pra opção: 19,4 

 

Isto é,  C=((H - K)/(H - L))*(S - L/(1+i))= 19,444 

 

Com H=200 

K=150 

L=50 

S=100 

  E tx de juros= 0,20 

 

Abraço 

 

De: [EMAIL PROTECTED] [mailto:[EMAIL PROTECTED] Em nome de Claudio Buffara
Enviada em: sábado, 5 de novembro de 2005 10:32
Para: obm-l@mat.puc-rio.br
Assunto: Re: RES: [obm-l] Economia na lista obm-l 
 

OK. Entao aqui vai, jah com desculpas pelo meio-off-topic.

Consideremos o caso de uma opcao de compra com 1 ano de prazo, preco de exercicio = K, sobre um ativo que hoje vale S e, daqui a um ano, vai valer:
H, com probabilidade p ou L, com probabilidade 1-p (L  H)

Isso significa que a opcao dah a seu titular o direito de adquirir o ativo pelo preco K dentro de 1 ano - o caso de interesse eh, naturalmente, quando L  K  H. 
Ou seja, se daqui a um ano o ativo valer H, o titular receberah H - K (ele exercerah a opcao, adquirindo o ativo por K e, imediatamente, poderah vender o ativo no mercado, recebendo H - se ele resolver nao ficar com o ativo, ele estrarah correndo um outro risco, o qual nao tem nada a ver com a opcao). Mas se o ativo valer H, ele nao receberah nem pagarah nada. 

Para nao haver arbitragem (ou seja, lucro garantido com risco zero - algo que nao pode acontecer num mercado verdadeiramente eficiente, coisa que nenhum eh de fato!), a seguinte relacao deve ser satisfeita: L  S(1+i)  H, onde i = taxa de juros (suposta constante ao longo do ano). Pergunta pra voce: por que essa relacao deve valer?

Nesse caso, talvez o mais surpreendente eh que o valor da opcao nao depende de p.
O que depende de p, dados H e L, eh justamente o preco a vista S.
Supondo que o mercado eh avesso a risco (o que me parece razoavel), a seguinte relacao deve prevalecer:
S  (H*p + L*(1-p))/(1+i), de modo que a rentabilidade esperada do ativo serah:
(H*p + L*(1-p))/S - 1  i = taxa de juros de uma aplicacao sem risco

No entanto, o mercado, se for eficiente, soh exige premio de risco (ou seja, uma rentabilidade acima da taxa de juros sem risco) de um dado ativo quando este risco nao for diversificavel.

No caso das opcoes, o risco eh totalmente diversificavel, uma vez que eh possivel construir uma carteira de investimentos composta do ativo-objeto da opcao e de um emprestimo, cujo fluxo de caixa eh exatamente igual ao do ativo. Logo, para nao haver arbitragem a carteira deve valer a mesma coisa que a opcao.

Assim, um investidor que vende a opcao e compra esta carteira nao terah risco algum e, portanto, nao deveria ter lucro algum.

O valor da opcao eh facil de calcular:

Na data inicial, o investidor vende a opcao de compra, arrecadando C, toma um emprestimo de B reais a juro i, e compra n unidades do ativo-objeto.
Logo, seu fluxo de caixa eh igual a C + B - n*S

Na data final: 
1) se o ativo valer H, o investidor pagarah H - K ao comprador da opcao, B*(1+i) ao banco, e receberah n*H pelo ativo

2) se o ativo valer L, o investidor nao pagarah nada ao comprador da opcao, pagarah B*(1+i) ao banco e receberah n*L pelo ativo.
(estou supondo que L  K  H)

Se quisermos zerar o fluxo de caixa na data final, teremos que escolher n e B de modo que:
n*H - (H-K) - B*(1+i) = 0
e
n*L - B*(1+i) = 0.

Resolvendo para n e B, obtemos:
n = (H - K)/(H - L) e B = ((H - K)/(H - L))*L/(1+i)

Se o fluxo de caixa no fim for zero em qualquer cenario, entao o fluxo de caixa inicial serah tambem 0, ou seja, dados n e B solucoes do sistema acima, teremos: 
C = n*S - B = ((H - K)/(H - L))*(S - L/(1+i))

[]s,
Claudio.

on 05.11.05 03:29, José Diogo Barbosa at [EMAIL PROTECTED] wrote: 

Olá cláudio 

Gostaria de ver a resposta certa desse problema! Se puder manda pra gente. Acho a resposta do artur muito boa também. Do arthur muito boa também! Se vc puder mostrar onde errei, vou agradecer 

ps: quase nunca me manifesto9 na lista, mas fico acampando e e sei a sua importância. 

Abraços 



 







Re: [obm-l] Novo na lista

2005-11-04 Por tôpico Claudio Buffara
Title: Re: [obm-l] Novo na lista



Se voce nao entendeu do jeito que estah abaixo, entao acho que uma inducao formal nao vai ajudar...

on 04.11.05 00:48, Adélman de Barros Villa Neto at [EMAIL PROTECTED] wrote:

Claudio Buffara:já que basta substituir por uma indução formal,pq você mesmo não substituiu?É exatamente isso que eu quero. 



On Wed, 02 Nov 2005 22:02:20 -0200, Aldo Munhoz escreveu:

 De: Aldo Munhoz 
 Data: Wed, 02 Nov 2005 22:02:20 -0200
 Para: obm-l@mat.puc-rio.br
 Assunto: Re: [obm-l] Novo na lista
 
 Vejamos um exemplo, seja n=59325. Separamos o digito5 das unidades e do numero restante 5932, subtraímos o dobro destedígito, isto é:


5932-10=5922


Em seguida repetimos este procedimento até aobtençao de um número suficientemente pequeno que possamos reconhecer,facilmente, se é ou não divisivel por 7.


592-4=588
58-16=42


Como 42 é divisivel por 7, o criterio que vamosprovar é que este fato irá implicar que o numero original também deveraser divisivel por 7.
Seja i o digito das unidades do numero n, entao n pode ser escrito como10k+i. (No exemplo acima k=5932 e i=5). No procedimento descrito acimaobtivemos um numero r como sendo k-2i. Feitas estas observacoes, serasuficiente provar que os numeros 10k+i e k-2i sao tais que, se um delesé multiplo de 7, o outro também é. Isto é, devemos provar a seguinteequivalencia:
10k+i é multiplo de 7 see k-2i é multiplo de 7.

Demonstração: (=) Se 10k+i é multiplo de 7, entao existe uminteiro m tal que 10k+i=7m e, portanto,k-2i=k-2(7m-10k)=k-14m+20k=21k-14m=7(3k-2m) o que imploca k-2i sermultiplo de 7.
(


No exemplo acima, como 42 é div! isivel por 7, entao 588 também é. Sendo588 divisivel por 7, e! ntao 5932 também devera ser e, a divisibilidadedeste por 7 implica que 59325 devera ser divisivel por 7.

Acho que isto prova o que você queria.

Abraços,

Aldo

Claudio Buffara wrote:
 Po, amigo! A demonstracao estah essencialmente completa. Basta notar que10^6 == 1 (mod 7) e, portanto, a coisa toda se repete com periodo 6 noexpoente de 10. Aquele E por ai vai... soh precisa ser substituido por umainducao formal, mas pra bom entenddor 99% de palavra deveria bastar.[]s,Claudio.on 02.11.05 15:38, Adélman de Barros Villa Neto at [EMAIL PROTECTED]: 
 ninguem ainda?On Mon, 31 Oct 2005 23:14:38 -0200, Adélman de Barros Villa Neto escreveu: 
 De: Adélman de Barros Villa Neto Data: Mon, 31 Oct 2005 23:14:38 -0200Para: [EMAIL PROTECTED]: [obm-l] Novo na listaOlá,estou procurando de um arquivo da lista onde é demonstrado um critério dedivisibilidade por 7.Alguem pode me ajudar?Encontrei essas mensagens mas emnem uma o autor completa a demonstração.Grato.Mod 7:1 == 110 == 3100 == 2 == (abc) = 100a + 10b + c == 2a + 3b + c (mod 7)Logo, 7 divide (abc) 7 divide 2a + 3b + c1000 == -11 == -310 == -2 ==(abcdef) = 10a + 1b + 1000c + 100d + 10e + f ==-2a -3b -c + 2d + 3e + f == -(2a+3b+c) + (2d+3e+f) (mod 7)Logo, 7 divide (abcdef) 7 divide -(2a+3b+c) + (2d+3e+f)E por ai vaiFicou claro?Entao farelo pra voce tambem.[]s,Claudio. ! 
 
 ===!
==Instruções para entrar na lista, sair da lista e usar a lista emhttp://www.mat.puc-rio.br/~nicolau/olimp/obm-l.html= 
=Instruções para entrar na lista, sair da lista e usar a lista emhttp://www.mat.puc-rio.br/~nicolau/olimp/obm-l.html= = Instruções para entrar na lista, sair da lista e usar a lista em http://www.mat.puc-rio.br/~nicolau/olimp/obm-l.html = 







Re: [obm-l] matrizes (olimpiada)

2005-11-04 Por tôpico Claudio Buffara
Title: Re: [obm-l] matrizes (olimpiada)



Voce soh pode fazer isso se souber de antemao que A e B sao invertiveis.

Por exemplo, A = B = matriz nula == AB = A e BA = B, mas A^2 + B^2  2I.

Sem maiores informacoes, acho que o maximo que dah pra concluir eh que A^2 + B^2 = A + B.

[]s,
Claudio.

on 04.11.05 09:15, Aldo Munhoz at [EMAIL PROTECTED] wrote:

AxB=A = A^(-1)xAxB=A^(-1)xA = B=I = B^2=I
BxA=B = B^(-1)xBxA=B^(-1)xB = A=I = A^2=I

Logo A^2+B^2=2I

Marcelo de Oliveira Andrade wrote: 
essa eh de uma olimpiada, esta na lista que o meu professor passou... 

AxB=A and BxA= B, A^2+B^2=? 

obrigado pela ajuda 







Re: [obm-l] matrizes (olimpiada)

2005-11-04 Por tôpico Claudio Buffara
Title: Re: [obm-l] matrizes (olimpiada)



AB = A == B(AB) = BA == (BA)B = BA == B^2 = B (pois BA = B)
Analogamente voce conclui que A^2 = A. Logo...
 
on 04.11.05 16:24, Aldo Munhoz at [EMAIL PROTECTED] wrote:

Claudio, não entendi como vc concluiu que A^2 + B^2 = A + B
Pode explicar melhor?
Abraços,
Aldo

Claudio Buffara wrote: 
Re: [obm-l] matrizes (olimpiada) Voce soh pode fazer isso se souber de antemao que A e B sao invertiveis.

Por exemplo, A = B = matriz nula == AB = A e BA = B, mas A^2 + B^2  2I.

Sem maiores informacoes, acho que o maximo que dah pra concluir eh que A^2 + B^2 = A + B.

[]s,
Claudio.

on 04.11.05 09:15, Aldo Munhoz at [EMAIL PROTECTED] wrote:

AxB=A = A^(-1)xAxB=A^(-1)xA = B=I = B^2=I
BxA=B = B^(-1)xBxA=B^(-1)xB = A=I = A^2=I

Logo A^2+B^2=2I

Marcelo de Oliveira Andrade wrote: 
essa eh de uma olimpiada, esta na lista que o meu professor passou... 

AxB=A and BxA= B, A^2+B^2=? 

obrigado pela ajuda 

= Instruções para entrar na lista, sair da lista e usar a lista em http://www.mat.puc-rio.br/~nicolau/olimp/obm-l.html = 







Re: [obm-l] Fatorar!Primalidade!Função Discreta!Tudo se realaciona?

2005-11-03 Por tôpico Claudio Buffara
Pelo que eu entendi, voce tem uma expressao do tipo:
y = (ax + b)/(cx + d), com a, b, c, d inteiros conhecidos e quer saber se
existe algum inteiro positivo x tal que y seja inteiro positivo.

Se esse for o caso, faca o seguinte:

Caso 1: c = 0.
Nesse caso, d tem que dividir ax + b, ou seja, ax == -b (mod d).
Essa congruencia soh terah solucao se mdc(a,d) dividir b.
Em caso afirmativo, a solucao no intervalo [1,d] serah unica.
Chame-a de x_0.
O conjunto-solucao serah:
{x_0 + md | m = 0, a(x_0 + md)/c  0 e ambos sao inteiros}

Caso 2: c  0 .
Re-escreva a expressao como y = (1/c)*(a + (bc - ad)/(cx + d)).
Pra y ser inteiro positivo, eh necessario que cx + d divida bc - ad.
Assim, teste os valores inteiros de x no intervalo [1,b-(a+1)d/c]
(se b-(a+1)d/c  1, entao nao existe solucao).
De qualqyer forma, o numero de candidatos a solucao serah finito.
Para cada x candidato, teste pra ver se c divide (a + (bc-ad)/(cx+d)).
Se algum divir e o quociente for positivo, voce teha achado o y
correspondente.

[]s,
Claudio.

on 02.11.05 15:21, Lestat di Lioncourt at [EMAIL PROTECTED] wrote:

 Bem eu tô com uma relação interessante que dá para associar a
 primalidade de um número, a fatoração de um número qualquer...
 acho que não é nada especial
 Mas tô pricisanu de uma ajuda!
 
 eu tô quereno analisar a seguinte função...
 
 
 y=  (c1 -10*x*c2)/(100*x+10c3)
 
 
 é o seguinte...eu tenho c1,c2,c3 mas não x...nem y
 logo é uma função em x e y
 Quero saber quando essa função vai ter um y natural para um x também
 natura
 
 Tem algum procedimento, raciocínio, fórmula para dizer se essa função
 tem par (x,y) natural?
 
 Ou alguma função que retorne se uma função tem par (x,y) natural num
 intervalo qualquer?
 
 Obirgado pela atenção!
 Vocês não sabem que grande ajuda seria!!!
 Por favor respondam!!!
 Brigadão!
 


=
Instruções para entrar na lista, sair da lista e usar a lista em
http://www.mat.puc-rio.br/~nicolau/olimp/obm-l.html
=


Re: [obm-l] Fatorar!Primalidade!Função Discreta!Tudo se realaciona?

2005-11-03 Por tôpico Claudio Buffara
Uma correcao:

No Caso 1, se mdc(a,d) dividir b, entao a solucao serah unica no intervalo
[1,d/mdc(a,d)].

on 03.11.05 07:08, Claudio Buffara at [EMAIL PROTECTED] wrote:

 Pelo que eu entendi, voce tem uma expressao do tipo:
 y = (ax + b)/(cx + d), com a, b, c, d inteiros conhecidos e quer saber se
 existe algum inteiro positivo x tal que y seja inteiro positivo.
 
 Se esse for o caso, faca o seguinte:
 
 Caso 1: c = 0.
 Nesse caso, d tem que dividir ax + b, ou seja, ax == -b (mod d).
 Essa congruencia soh terah solucao se mdc(a,d) dividir b.
 Em caso afirmativo, a solucao no intervalo [1,d] serah unica.
 Chame-a de x_0.
 O conjunto-solucao serah:
 {x_0 + md | m = 0, a(x_0 + md)/c  0 e ambos sao inteiros}
 
 Caso 2: c  0 .
 Re-escreva a expressao como y = (1/c)*(a + (bc - ad)/(cx + d)).
 Pra y ser inteiro positivo, eh necessario que cx + d divida bc - ad.
 Assim, teste os valores inteiros de x no intervalo [1,b-(a+1)d/c]
 (se b-(a+1)d/c  1, entao nao existe solucao).
 De qualqyer forma, o numero de candidatos a solucao serah finito.
 Para cada x candidato, teste pra ver se c divide (a + (bc-ad)/(cx+d)).
 Se algum divir e o quociente for positivo, voce teha achado o y
 correspondente.
 
 []s,
 Claudio.
 
 on 02.11.05 15:21, Lestat di Lioncourt at [EMAIL PROTECTED] wrote:
 
 Bem eu tô com uma relação interessante que dá para associar a
 primalidade de um número, a fatoração de um número qualquer...
 acho que não é nada especial
 Mas tô pricisanu de uma ajuda!
 
 eu tô quereno analisar a seguinte função...
 
 
 y=  (c1 -10*x*c2)/(100*x+10c3)
 
 
 é o seguinte...eu tenho c1,c2,c3 mas não x...nem y
 logo é uma função em x e y
 Quero saber quando essa função vai ter um y natural para um x também
 natura
 
 Tem algum procedimento, raciocínio, fórmula para dizer se essa função
 tem par (x,y) natural?
 
 Ou alguma função que retorne se uma função tem par (x,y) natural num
 intervalo qualquer?
 
 Obirgado pela atenção!
 Vocês não sabem que grande ajuda seria!!!
 Por favor respondam!!!
 Brigadão!
 
 
 
 =
 Instruções para entrar na lista, sair da lista e usar a lista em
 http://www.mat.puc-rio.br/~nicolau/olimp/obm-l.html
 =
 


=
Instruções para entrar na lista, sair da lista e usar a lista em
http://www.mat.puc-rio.br/~nicolau/olimp/obm-l.html
=


Re: [obm-l] DIVERSÃO PROBABILÍSTICA!

2005-11-03 Por tôpico Claudio Buffara
on 03.11.05 11:27, Jorge Luis Rodrigues e Silva Luis at
[EMAIL PROTECTED] wrote:

 Turma! Gostaria de dedicar esse singelo artigo ao amigo postal Chicão
 Valadares, por três motivos pessoais: primeiro, por ter sido o único que
 encaminhou minhas mensagens à lista na ocasião do meu afastamento; segundo,
 por ter veiculado meu retorno à mesma e terceiro, por ser um assunto que faz
 parte da sua praia estatística.
 
 Existem bolas azuis e vermelhas em uma caixa. A probabilidade de sortear
 duas bolas de cores diferentes, ao retirar duas bolas ao acaso, é 1/2. Prove
 que o número de bolas na caixa é um quadrado perfeito.
 
a bolas azuis e v bolas vermelhas na caixa.
Bolas de mesma cor sao supostas indistinguiveis.

2 bolas de cores distintas podem ser retiradas de 2av maneiras
2 bolas podem ser retiradas de (a+v)(a+v-1) maneiras

Prob(2 bolas de cores distintas) = 2av/((a+v)(a+v-1)) = 1/2 ==
a^2 + v^2 + 2av - a - v = 4av ==
(a - v)^2 = a + v ==
no. de bolas na caixa = a + v = quadrado perfeito
  
 Um jogador lança uma moeda não viciada e marca um ponto cada vez que obtém
 uma cara e dois pontos quando obtém coroa. Qual a probabilidade do jogador
 marcar exatamente n pontos?

P(n) = P(n|n-1)*P(n-1) + P(n|n-2)*P(n-2) = (1/2)*P(n-1) + (1/2)*P(n-2)
P(1) = 1/2  e  P(2) = 1/2 + 1/4 = 3/4

Eq. caracteristica: t^2 = (t+1)/2 == 2t^2 - t - 1 = 0 ==
t = 1  ou  t = -1/2 ==
P(n) = A + B*(-1/2)^n

P(1) = A - B/2 = 1/2
P(2) = A + B/4 = 3/4 ==

A = 2/3   B = 1/3 ==

P(n) = 2/3 + (1/3)*(-1/2)^n
 

 Jogamos 10 dados comuns (com 6 faces equiprováveis numeradas de 1 a 6).
 Calcule a probabilidade de que a soma dos 10 resultados seja igual a 20.
 
A probabilidade eh igual a N/6^10, onde:
N = coeficiente de x^20 na expansao de
(x + x^2 + x^3 + x^4 + x^5 + x^6)^10

 
 Guilherme lançou uma moeda quatro vezes. A probabilidade de ele obter no
 mínimo tantas caras quanto coroas é ?

Supondo a moeda honesta, P(2, 3 ou 4 caras) = (6+4+1)/2^4 = 11/16.


 A propósito, quantos dados devem ser lançados ao mesmo tempo para maximizar
 a probabilidade de se obter exatamente um 2?
 
Lancemos N dados honestos.

Escolha do dado que vai dar 2: N
Escolha dos resultados dos outros N-1 dados: 5^(N-1)

Probabilidade = f(N) = N*5^(N-1)/6^N =

f'(N) = (5^(N-1)/6^N)*(1 + N*log(5/6)) = 0 ==
N = 1/log(6/5) ~ 5,48

N = 5 e N = 6 dao a mesma probabilidade maxima, igual a (5/6)^5.


Acho que eh isso.


[]s,
Claudio.


=
Instruções para entrar na lista, sair da lista e usar a lista em
http://www.mat.puc-rio.br/~nicolau/olimp/obm-l.html
=


Re: [obm-l] DIVERSÃO PROBABILÍSTICA!

2005-11-03 Por tôpico Claudio Buffara
Title: Re: [obm-l] DIVERSÃO PROBABILÍSTICA!



O que eu calculei (ou acho que calculei!) foi a probabilidade de que uma dada sequencia crescente de numeros gerados pelo lancamento da moeda contenha o numero n. Fiz isso porque o enunciado nao dava nenhuma dica de haver uma ordem temporal no problema. Mas admito que haja outras interpretacoes.

[]s,
Claudio.

on 03.11.05 16:32, leonardo maia at [EMAIL PROTECTED] wrote:

Claudio, é preciso introduzir uma segunda variável t (o tempo, discreto) para que a questão

 Um jogador lança uma moeda não viciada e marca um ponto cada vez que obtém
 uma cara e dois pontos quando obtém coroa. Qual a probabilidade do jogador
 marcar exatamente n pontos?

faça sentido. Você resolveu a recorrência

P(n) = (1/2)*P(n-1) + (1/2)*P(n-2)

como se essas probabilidades fossem definidas no mesmo instante, o que não é verdade. A equação correta é

P(n,t) = (1/2)*P(n-1, t-1) + (1/2)*P(n-2, t-1).

Se você introduzir uma função geradora (ou geratriz) g, pode obter uma recorrência (em t) para g, resolvê-la, e descobrir que P(n,t) é zero, se nt ou n2t, e que, para t = n = 2t, vale

P(n,t) = [(1/2)^t] . C(t, n-t).

[]'s, Leo.


On 11/3/05, Claudio Buffara [EMAIL PROTECTED] wrote:
on 03.11.05 11:27, Jorge Luis Rodrigues e Silva Luis at
[EMAIL PROTECTED] wrote:

 Um jogador lança uma moeda não viciada e marca um ponto cada vez que obtém
 uma cara e dois pontos quando obtém coroa. Qual a probabilidade do jogador
 marcar exatamente n pontos?

P(n) = P(n|n-1)*P(n-1) + P(n|n-2)*P(n-2) = (1/2)*P(n-1) + (1/2)*P(n-2)
P(1) = 1/2 e P(2) = 1/2 + 1/4 = 3/4

Eq. caracteristica: t^2 = (t+1)/2 == 2t^2 - t - 1 = 0 ==
t = 1 ou t = -1/2 == 
P(n) = A + B*(-1/2)^n

P(1) = A - B/2 = 1/2
P(2) = A + B/4 = 3/4 ==

A = 2/3 B = 1/3 ==

P(n) = 2/3 + (1/3)*(-1/2)^n

\





Re: [obm-l] para Claudio( n° complexos)

2005-11-03 Por tôpico Claudio Buffara
Title: Re: [obm-l] para Claudio( n° complexos)



Eh isso mesmo. Voce poderia tentar usar esse metodo geometrico (e compara-lo com o algebrico) pra resolver os quatro problemas abaixo.

Achar os complexos z tais que:
1) |(z-i)/(z-1)| = a, para a  0 e a  1.
2) |z-i| + |z-1| = b (para que valores de b este conjunto eh nao vazio?)
3) |z-i| - |z-1| = c
4) |(z-i)*(z-1)| = d
 
[]s,
Claudio.

on 03.11.05 20:02, gustavo at [EMAIL PROTECTED] wrote:

 

Oi Claudio, então a resposta seria apenas a condição a =b, para qualquer valor real.obrigado pela sua opinão. 
 
- Original Message - 
From: claudio.buffara mailto:[EMAIL PROTECTED] 
To: obm-l mailto:obm-l@mat.puc-rio.br 
Sent: Tuesday, November 01, 2005 4:12 PM
Subject: [obm-l] Re:[obm-l] Re: [obm-l] Número Complexo

Outra forma de resolver o problema é observar que, no plano complexo, o lugar geométrico dos complexos z tais que:
|(z-i)/(z-1)| = 1 == |z-i| = |z-1| e z  1
é a mediatriz do segmento cujas extremidades são os complexos 1 e i, ou seja, a reta Re(z) = Im(z), bissetriz dos quadrantes ímpares.
 
[]s,
Claudio.
 
De: [EMAIL PROTECTED]
Para: obm-l@mat.puc-rio.br
Cópia: 
Data: Tue, 1 Nov 2005 13:42:04 -0200
Assunto: [obm-l] Re: [obm-l] Número Complexo
 Bom, resolvendo aqui também encontrei a=b.
 Logo, qualquer a e b satisfazem a equacao, inclusive a = b = 0.
 
 Abraço,
 Marcelo
- Original Message - 
From: gustavo mailto:[EMAIL PROTECTED] 
To: obm-l@mat.puc-rio.br 
Sent: Monday, October 31, 2005 6:45 PM
Subject: [obm-l] Número Complexo
 
 Sendo Z = a + bi, e I (Z - i) / ( Z - 1) I = 1 , ou seja o módulo deste quociente é igual 1. Encontrei apenas que a =b porém o gabarito que eu tenho informa que a =b = 1/2 ,porém testei a=b = 3 , deu certo !! em tempo: não seria a = b e diferenre de zero ,desde já agradeço qualquer ajuda !! 

No virus found in this incoming message.
Checked by AVG Free Edition.
Version: 7.1.362 / Virus Database: 267.12.6/152 - Release Date: 31/10/05


No virus found in this incoming message.
Checked by AVG Free Edition.
Version: 7.1.362 / Virus Database: 267.12.6/152 - Release Date: 31/10/05







Re: [obm-l] equacao3

2005-11-02 Por tôpico Claudio Buffara
Title: Re: [obm-l] equacao3



De onde voce estah tirando estes problemas?

Qualquer bom livro de teoria dos numeros ou teoria dos numeros algebricos descreve pelo menos os metodos de solucao dessas equacoes.

[]s,
Claudio.

on 02.11.05 10:26, Klaus Ferraz at [EMAIL PROTECTED] wrote:

Mostre que a equacao x2 + 2 =y3 tem apenas uma solução inteira positiva.






Re: [obm-l] duvidas - recorrencia e somatorio

2005-11-02 Por tôpico Claudio Buffara
on 02.11.05 14:37, Guilherme Augusto at [EMAIL PROTECTED] wrote:

 Tenho algumas duvidas e gostaria que voces da lista me ajudassem.
 
 1) quando eu tenho em uma equação característica de
 uma recorrência, do tipo a_(n)*t^n +
 a_(n-1)*t^(n-1)+...+ a_0=0 e encontro dois (ou
 mais)resultados iguais para t, o que eu faço? E quando
 uma das soluções em t é 1?
 
Um fator da forma (x-r)^k no polinomio caracteristico dah origem a um termo
da forma (b_0 + b_1*n + b_2*n^2 + ... + b_(k-1)*n^(k-1))*r^k na formula de
a_n. Isso vale mesmo quando r = 1.

 2) como eu resolvo Soma(1, infinito)(1/i^2) sem
 recorrer a cálculo? Onde eu peguei dizia que era
 possível usando apenas propriedades de somatório. (na
 verdade, pedia para provar que a soma é (pi^2)/6 )

Sem usar calculo deve ser complicado pois Pi eh definido rigorosamente
usando o calculo (mais precisamente, as funcoes seno e cosseno sao definidas
como certas series de potencias e Pi/2 eh definido como sendo a menor raiz
positiva da funcao cosseno). Alem disso, o samatorio eh infinito, o que
envolve o conceito de limite.

Agora, se voce estah interessado neste tipo de coisa, recomendo que voce
comece a estudar calculo (mesmo que nao faca parte do curriculo do ensino
medio), pois varios problemas, cujas solucoes por metodos elementares sao
muito dificeis ou ateh impossiveis, ficam triviais com o uso do calculo.
Por exemplo, qual a area da regiao delimitada pelo eixo x, as retas x = 0 e
x = 1 e a parabola y = x^2?


[]s,
Claudio.


=
Instruções para entrar na lista, sair da lista e usar a lista em
http://www.mat.puc-rio.br/~nicolau/olimp/obm-l.html
=


Re: [obm-l] equacao3

2005-11-02 Por tôpico Claudio Buffara
Title: Re: [obm-l] equacao3



Mas foi isso que eu quis dizer: pegue um bom livro de teoria dos numeros e estude.

x^2 + 2 = y^3 pode ser resolvida atraves do uso de propriedades do dominio euclidiano Z[raiz(-2)]. 

x^2 - Ay^2 = 1 eh chamada equacao de Pell (ou Pell-Fermat) e pode ser resolvida por meio de fracoes continuas.

O que sao um dominio euclidiano e uma fracao continua e que eles tem a ver com estas equacoes? Pegue o livro e leia.

Ou se voce quiser solucoes mais elementares, de uma olhada em:
An Adventurers Guide to Number Theory de Richard Friedberg - cap. 7 - para a cubica
e no excelente 100 Great Problems of Elementary Mathematics de Heinrich Dorrie - cap. 20 - para a equacao de Pell.


[]s,
Claudio.

on 02.11.05 14:24, Adroaldo Munhoz at [EMAIL PROTECTED] wrote:

Como resolve?


Claudio Buffara wrote: 
Re: [obm-l] equacao3 De onde voce estah tirando estes problemas?

Qualquer bom livro de teoria dos numeros ou teoria dos numeros algebricos descreve pelo menos os metodos de solucao dessas equacoes.

[]s,
Claudio.

on 02.11.05 10:26, Klaus Ferraz at [EMAIL PROTECTED] wrote:

Mostre que a equacao x2 + 2 =y3 tem apenas uma solução inteira positiva.






<    2   3   4   5   6   7   8   9   10   11   >